Diagnostic différentiel des maladies bénignes et malignes de la vésicule biliaire et des voies biliaires extrahépatiques. Cholélithiase Diff diagnostic des maladies des voies biliaires

La lithiase biliaire (GSD) est une maladie caractérisée par la formation de calculs dans la vésicule biliaire (cholécystolithiase) et dans le canal cholédoque (cholédocholithiase), qui peut survenir avec des symptômes de colique biliaire (biliaire, hépatique) en réponse à une obstruction transitoire des voies kystique ou hépatique. voie cholédoque par une pierre, accompagnée d'un spasme des muscles lisses et d'une hypertension intracanalaire.

Entre 21 et 30 ans, 3,8% de la population souffre de lithiase biliaire, de 41 à 50 ans - 5,25%, de plus de 60 ans - jusqu'à 20%, de plus de 70 ans - jusqu'à 30%. Le sexe prédominant est le sexe féminin (3–5:1), bien qu'il y ait une tendance à une augmentation de l'incidence chez les hommes.

Facteurs prédisposant à la formation de calculs biliaires (cholestérol principalement) : sexe féminin ; l'âge (plus le patient est âgé, plus le risque de lithiase biliaire est élevé); caractéristiques génétiques et ethniques; nature des aliments surutilisation aliments gras riches en cholestérol, graisses animales, sucre, sucreries ; grossesse (naissances multiples dans l'histoire); obésité; famine; zones géographiques de résidence; maladies de l'iléon - syndrome de l'intestin court, maladie de Crohn, etc.; l'utilisation de certains médicaments - œstrogènes, octréotide, etc.

Classification

1. Par la nature des pierres

1.1 Composition : cholestérol ; pigment; mixte.

1.2 Par localisation : dans la vésicule biliaire ; dans le canal cholédoque (cholédocholithiase); dans les conduits hépatiques.

1.3 Par le nombre de pierres : simple ; plusieurs.

2. Selon l'évolution clinique

2.1 parcours latent ;

2.2 avec présence de symptômes cliniques : forme de douleur avec colique biliaire typique ; forme dyspeptique; sous couvert d'autres maladies.

3. Complications : cholécystite aiguë; hydropisie de la vésicule biliaire; cholédocholithiase; ictère mécanique; pancréatite aiguë; cholangite purulente; fistules bilieuses; rétrécissement de la grande papille duodénale.

Image clinique

Souvent, la lithiase biliaire est asymptomatique (évolution latente, caractéristique de 75% des patients) et les calculs sont découverts par hasard lors d'une échographie. Le diagnostic de lithiase biliaire est posé sur la base des données cliniques et des résultats échographiques. La variante la plus courante est la colique biliaire : elle survient chez 60 à 80 % des personnes atteintes de calculs biliaires et chez 10 à 20 % des personnes atteintes de calculs du cholédoque.

La principale manifestation clinique de la lithiase biliaire est la colique biliaire.Elle se caractérise par une douleur viscérale aiguë localisée dans l'épigastre ou l'hypochondre droit, moins souvent la douleur ne survient que dans l'hypochondre gauche, la région précordiale ou le bas-ventre, ce qui complique considérablement le diagnostic. Chez 50% des patients, la douleur irradie vers le dos et l'omoplate droite, la région interscapulaire, l'épaule droite, moins souvent vers la moitié gauche du corps. La durée des coliques biliaires varie de 15 minutes à 5-6 heures. Une douleur qui dure plus de 5 à 6 heures doit alerter le médecin de l'ajout de complications, principalement cholécystite aiguë. Pour syndrome douloureux caractérisé par une transpiration accrue, une grimace de douleur sur le visage et un comportement agité du patient. Parfois, des nausées et des vomissements surviennent. L'apparition de douleurs peut être précédée par l'utilisation d'aliments gras, épicés, épicés, d'alcool, d'activité physique, d'expériences émotionnelles. La douleur est associée à une hyperdistension de la paroi de la vésicule biliaire due à une augmentation de la pression intravésicale et à une contraction spasmodique du sphincter d'Oddi ou du canal cystique. Avec les coliques biliaires, la température corporelle est généralement normale, la présence d'hyperthermie associée à des symptômes d'intoxication (tachycardie, sécheresse et langue velue) indique généralement l'ajout d'une cholécystite aiguë.

L'identification de la jaunisse est considérée comme un signe d'obstruction des voies biliaires.

Lors de la collecte d'une anamnèse, il est nécessaire d'interroger particulièrement soigneusement le patient sur les épisodes de douleurs abdominales dans le passé, car avec la progression de la maladie des calculs biliaires, les épisodes de coliques biliaires se reproduisent, se prolongent et l'intensité de la douleur augmente.

Des symptômes non spécifiques sont également possibles, tels qu'une lourdeur dans l'hypochondre droit, des manifestations de dyskinésie biliaire, des flatulences, des troubles dyspeptiques.

Un examen objectif peut révéler des symptômes de cholécystite chronique (symptômes vésicaux). JE SUIS AVEC. Zimmerman (1992) a systématisé les symptômes physiques de la cholécystite chronique en trois groupes comme suit.

Symptômes du premier groupe (symptômes réflexes segmentaires) sont causés par une irritation prolongée des formations segmentaires du système nerveux autonome qui innervent le système biliaire, et sont divisés en deux sous-groupes :

1. Points et zones réflexes douloureux viscérocutanés- caractérisé par le fait que la pression des doigts sur des points spécifiques d'organes de la peau provoque des douleurs :

la douleur Pointe du Mackenzie situé à l'intersection du bord externe du muscle droit de l'abdomen avec l'arc costal droit ;

la douleur Pointe des boas-situé sur la face postérieure poitrine le long de la ligne paravertébrale à droite au niveau des vertèbres thoraciques X-XI ;

zones d'hypertension cutanée Zakharyin-Ged- de vastes zones de douleur intense et d'hypersensibilité, s'étendant dans toutes les directions à partir des points Mackenzie et Boas.

2. Symptômes réflexes cutanéo-viscéraux- se caractérisent par le fait que l'impact sur certains points ou zones provoque des douleurs s'approfondissant vers la vésicule biliaire :

Le symptôme d'Aliev la pression sur les points Mackenzie ou Boas provoque non seulement une douleur locale directement sous le doigt qui palpe, mais aussi une douleur pénétrant profondément dans la vésicule biliaire ;

Le symptôme d'Eisenberg-I avec un coup court ou en tapotant avec le bord de la paume sous l'angle de l'omoplate droite, le patient, accompagné d'une douleur locale, ressent une irradiation prononcée profondément dans la région de la vésicule biliaire.

Les symptômes du premier groupe sont naturels et caractéristiques de l'exacerbation de la cholécystite chronique. Les plus pathognomoniques sont les symptômes de Mackenzie, Boas, Aliev.

Symptômes du deuxième groupe en raison de la propagation de l'irritation du système nerveux autonome au-delà de l'innervation segmentaire du système biliaire à toute la moitié droite du corps et des membres droits. Dans ce cas, un syndrome végétatif réactif du côté droit se forme, caractérisé par l'apparition de douleurs lors de la palpation des points suivants:

Point orbital de Bergmann(au bord intérieur supérieur de l'orbite);

point occipital de Yonash ;

Pointe Mussi-Georgievsky(entre les jambes du droit m. sternocleidomastoideus)

– symptôme phrénique droit ;

point interscapulaire de Kharitonov(au milieu d'une ligne horizontale passant par le milieu du bord intérieur de l'omoplate droite);

pointe fémorale de Lapinsky(milieu du bord intérieur de la cuisse droite);

pointe du creux poplité droit ;

pointe plantaire(à l'arrière du pied droit).

La pression sur les points indiqués est faite par la pointe du pointeur

doigt du corps. Les symptômes du deuxième groupe sont observés dans l'évolution souvent récurrente de la cholécystite chronique. La présence de douleur en même temps en plusieurs ou même plus en tous les points reflète la sévérité de l'évolution de la maladie.

Symptômes du troisième groupe sont détectés avec une irritation directe ou indirecte (par tapotement) de la vésicule biliaire (symptômes irritatifs). Ceux-ci inclus:

signe de Murphy pendant que le patient expire, le médecin plonge soigneusement le bout des quatre doigts à moitié pliés de la main droite sous l'arc costal droit dans la région de la vésicule biliaire, puis le patient respire profondément, le symptôme est considéré comme positif si, pendant l'expiration, le patient l'interrompt soudainement en raison de l'apparition d'une douleur lorsque le bout des doigts touche une vésicule biliaire enflammée sensible. En même temps, une grimace de douleur peut apparaître sur le visage du patient ;

Le symptôme de Kera- Douleur dans l'hypochondre droit dans la région de la vésicule biliaire palpation profonde;

Symptôme de Gausmann- l'apparition de douleurs avec un coup court avec le bord de la paume sous l'arc costal droit à la hauteur de l'inspiration) ;

symptôme de Lepene-Vasilenko- l'apparition de douleurs lors de l'application de coups saccadés du bout des doigts en inspirant sous l'arc costal droit ;

symptôme d'Ortner-Grekov- l'apparition de douleurs lors du tapotement de l'arc costal droit avec le bord de la paume (la douleur apparaît en raison de la commotion cérébrale enflammée);

Symptôme d'Eisenberg II- en position debout, le patient se lève sur ses orteils puis tombe rapidement sur ses talons, avec un symptôme positif, une douleur apparaît dans l'hypochondre droit en raison d'une commotion cérébrale enflammée.

Les symptômes du troisième groupe ont une grande valeur diagnostique, en particulier dans la phase de rémission, d'autant plus que dans cette phase les symptômes des deux premiers groupes sont généralement absents.

Symptômes d'implication dans le processus pathologique du plexus solaire

Avec un long cours de cholécystite chronique, une implication dans le processus pathologique du plexus solaire est possible - un syndrome solaire secondaire.

Les principaux signes du syndrome solaire sont :

Douleur dans la région ombilicale avec irradiation dans le dos (solaralgie), parfois la douleur est de nature brûlante ;

Phénomènes dyspeptiques (ils sont difficiles à distinguer des symptômes de dyspepsie dus à l'exacerbation de la cholécystite chronique elle-même et à la pathologie concomitante de l'estomac);

Détection palpatoire des points douloureux situés entre le nombril et le processus xiphoïde ;

Symptôme Pekarsky - douleur en appuyant sur le processus xiphoïde.

Diagnostique

Pour la lithiase biliaire non compliquée, les modifications des paramètres de laboratoire ne sont pas caractéristiques. Avec le développement d'une cholécystite aiguë et d'une cholangite concomitante, une leucocytose, une augmentation de la VS, une augmentation de l'activité des aminotransférases sériques, des enzymes de cholestase (phosphatase alcaline, gamma-glutamyl transpeptidase) et des taux de bilirubine sont possibles.

S'il existe une suspicion cliniquement justifiée de lithiase biliaire, une échographie est nécessaire en premier lieu. Le diagnostic de lithiase biliaire est confirmé par la TDM, la cholangiopancréatographie par résonance magnétique, la cholécystographie, la cholécystopancréaticographie endoscopique.

Études instrumentales obligatoires

■ Échographie d'organes cavité abdominale comme la méthode la plus accessible avec une sensibilité et une spécificité élevées pour la détection des calculs biliaires. Pour les calculs dans la vésicule biliaire et le canal cystique, la sensibilité des ultrasons est de 89 %, la spécificité est de 97 %, pour les calculs dans le canal cholédoque, la sensibilité est inférieure à 50 % et la spécificité est de 95 %. Une recherche ciblée est nécessaire : dilatation des voies biliaires intra- et extra-hépatiques ; calculs dans la lumière de la vésicule biliaire et des voies biliaires ; signes de cholécystite aiguë sous la forme d'un épaississement de la paroi vésiculaire de plus de 4 mm, révélant un "double contour" de la paroi vésiculaire.

■ Radiographie standard de la zone vésiculaire : la sensibilité de la méthode de détection des calculs est inférieure à 20 % en raison de leur fréquente négativité aux rayons X.

■ EGDS : réalisé pour évaluer l'état de l'estomac et du duodénum, ​​examen de la grosse papille du duodénum avec suspicion de cholédocholithiase.

Études instrumentales supplémentaires

■ Cholécystographie orale ou intraveineuse. Une vésicule biliaire «handicapée» (les voies biliaires extrahépatiques sont contrastées et la vessie n'est pas définie) peut être considérée comme un résultat significatif de l'étude, ce qui indique une oblitération ou un blocage du canal cystique.

■ scanner des organes abdominaux (vésicule biliaire, voies biliaires, foie, pancréas) avec détermination quantitative du coefficient d'atténuation des calculs biliaires selon Hansfeld ; la méthode permet de juger indirectement de la composition des pierres par leur densité.

■ Cholécystopancréaticographie endoscopique : méthode très informative pour étudier les voies extrahépatiques en cas de suspicion de calcul du cholédoque ou pour exclure d'autres maladies et causes d'ictère obstructif.

■ La cholescintigraphie dynamique permet d'évaluer la perméabilité des voies biliaires dans les cas où la cholécystopancréaticographie endoscopique est difficile. Chez les patients atteints de lithiase biliaire, une diminution du taux d'entrée du radiopharmaceutique dans la vésicule biliaire et les intestins est déterminée.

Diagnostic différentiel

Le syndrome douloureux dans la lithiase biliaire doit être différencié des conditions suivantes.

■ Boues biliaires : le tableau clinique typique de la colique biliaire est parfois observé. L'échographie révèle la présence d'une vésicule biliaire dans la vésicule biliaire.

■ Maladies fonctionnelles de la vésicule biliaire et des voies biliaires : l'examen ne retrouve pas de calculs. Détecter les signes d'altération de la contractilité de la vésicule biliaire (hypo- ou hyperkinésie), spasme de l'appareil sphinctérien (dysfonctionnement du sphincter d'Oddi).

■ Pathologie de l'œsophage : œsophagite, œsophagospasme, hernie hiatale. Caractérisé par une douleur dans la région épigastrique et derrière le sternum, associée à des modifications typiques de l'endoscopie ou de l'examen radiographique du tractus gastro-intestinal supérieur.

■ Ulcère peptique de l'estomac et du duodénum : caractérisé par des douleurs dans la région épigastrique, irradiant parfois vers le dos et diminuant après avoir mangé, pris des antiacides et des antisécrétoires. EGDS est requis.

■ Maladies du pancréas : pancréatites aiguës et chroniques, pseudokystes, tumeurs. Douleur typique de la région épigastrique, irradiant vers le dos, provoquée par l'alimentation et souvent accompagnée de vomissements. L'activité accrue de l'amylase et de la lipase dans le sérum sanguin, ainsi que les changements typiques selon les résultats des méthodes de radiodiagnostic, témoignent en faveur du diagnostic. Il convient de garder à l'esprit que la cholélithiase et les boues biliaires peuvent entraîner le développement d'une pancréatite aiguë.

■ Maladie du foie : typique Douleur contondante dans l'hypochondre droit, rayonnant vers le dos et l'omoplate droite. La douleur est généralement constante (ce qui n'est pas typique de la douleur dans les coliques biliaires), est associée à une augmentation du foie et une sensibilité du foie à la palpation est caractéristique.

■ Maladies du côlon : syndrome de l'intestin irritable, tumeurs, lésions inflammatoires (surtout lorsque l'angle hépatique du côlon est impliqué dans le processus pathologique). Le syndrome douloureux est souvent causé par des troubles moteurs. La douleur s'améliore souvent après une selle ou des flatulences. Pour le diagnostic différentiel des changements fonctionnels et organiques, une coloscopie ou un lavement baryté est recommandé.

■ Maladies des poumons et de la plèvre : une radiographie pulmonaire est nécessaire.

■ Pathologie des muscles squelettiques : douleur de l'hypochondre droit de l'abdomen associée à des mouvements ou à l'adoption d'une certaine position corporelle. La palpation des côtes peut être douloureuse ; une douleur accrue est possible avec une tension dans les muscles de la paroi abdominale antérieure.

Traitement

Objectifs de la thérapie :élimination des calculs biliaires (soit les calculs eux-mêmes des voies biliaires, soit la vésicule biliaire avec des calculs); soulagement des symptômes cliniques sans intervention chirurgicale (s'il existe des contre-indications au traitement chirurgical); prévention du développement de complications, tant immédiates (cholécystite aiguë, pancréatite aiguë, cholangite aiguë) qu'à distance (cancer de la vésicule biliaire).

Indications d'hospitalisation en hôpital chirurgical : coliques biliaires récurrentes ; pointu et cholécystite chronique s et leurs complications ; ictère mécanique; cholangite purulente; pancréatite aiguë biliaire.

Indications d'hospitalisation dans un hôpital gastro-entérologique ou thérapeutique: cholécystite calculeuse chronique - pour un examen détaillé et une préparation à un traitement chirurgical ou conservateur; exacerbation de lithiase biliaire et état après cholécystectomie (pancréatite biliaire chronique, dysfonctionnement du sphincter d'Oddi).

Durée du traitement hospitalier: cholécystite calculeuse chronique - 8-10 jours, pancréatite biliaire chronique (selon la gravité de la maladie) - 21-28 jours.

Le traitement comprend une thérapie diététique, des médicaments, une lithotritie extracorporelle et une intervention chirurgicale.

Thérapie diététique : à tous les stades, 4 à 6 repas par jour sont recommandés à l'exception des aliments qui augmentent la séparation de la bile, la sécrétion de l'estomac et du pancréas. Exclure les viandes fumées, les graisses réfractaires, les assaisonnements irritants. Le régime alimentaire doit inclure une grande quantité de fibres végétales avec l'ajout de son, ce qui non seulement normalise la motilité intestinale, mais réduit également la lithogénicité de la bile. Avec les coliques biliaires, le jeûne est nécessaire pendant 2-3 jours.

La litholyse orale est le seul traitement conservateur efficace de la lithiase biliaire. Médicaments utilisés pour dissoudre les calculs acides biliaires: acides ursodésoxycholique et chénodésoxycholique. Le traitement aux acides biliaires est réalisé et suivi en ambulatoire.

Les conditions les plus favorables à l'issue de la lithotripsie orale : stades précoces de la maladie ; cholélithiase non compliquée, épisodes rares de colique biliaire, syndrome douloureux modéré ; en présence de calculs de cholestérol pur ("flotteur" lors d'une cholécystographie orale) ; en présence de calculs non calcifiés (coefficient d'atténuation au CT inférieur à 70 unités Hansfeld) ; avec des tailles de pierre ne dépassant pas 15 mm (en combinaison avec la lithotritie par ondes de choc - jusqu'à 30 mm), les meilleurs résultats sont observés avec des diamètres de pierre jusqu'à 5 mm; avec des pierres simples n'occupant pas plus de 1/3 de la vésicule biliaire ; avec fonction contractile préservée de la vésicule biliaire.

Les doses quotidiennes de médicaments sont déterminées en tenant compte du poids corporel du patient. La dose d'acide chénodésoxycholique (sous forme de monothérapie) est de 15 mg/(kg jour), l'acide ursodésoxycholique (sous forme de monothérapie) est de 10 à 15 mg/(kg jour). Il faut privilégier les dérivés de l'acide ursodésoxycholique, car ils sont plus efficaces et ont moins d'effets secondaires. La combinaison d'acides ursodésoxycholique et chénodésoxycholique à une dose de 7 à 8 mg/(kg jour) de chaque médicament est considérée comme la plus efficace. Les médicaments sont prescrits une fois la nuit.

Le traitement est effectué sous contrôle échographique (1 fois en 3-6 mois). En présence d'une dynamique positive avec ultrasons, 3 à 6 mois après le début du traitement, celui-ci est poursuivi jusqu'à la dissolution complète des calculs. La durée du traitement varie généralement de 12 à 24 mois avec une utilisation continue de médicaments. Quelle que soit l'efficacité de la thérapie litholytique, elle réduit l'intensité de la douleur et réduit le risque de développer une cholécystite aiguë.

L'efficacité du traitement conservateur est assez élevée: avec une sélection appropriée des patients, une dissolution complète des calculs est observée après 18 à 24 mois chez 60 à 70% des patients, mais les rechutes de la maladie ne sont pas rares.

L'absence de dynamique positive selon les données échographiques après 6 mois de prise des médicaments indique l'inefficacité de la thérapie litholytique orale et indique la nécessité de l'arrêter.

Étant donné que le syndrome douloureux de la colique biliaire est davantage associé à un spasme de l'appareil sphinctérien, il est justifié de prescrire des antispasmodiques (mébévérine, bromure de pinavérium) à des doses quotidiennes standard pendant 2 à 4 semaines.

Le traitement antibactérien est indiqué pour la cholécystite aiguë et l'angiocholite.

Méthodes de traitement chirurgical: cholécystectomie - lithotripsie par onde de choc extracorporelle laparoscopique ou ouverte.

Indications du traitement chirurgical de la cholécystolithiase : la présence de gros et petits calculs dans la vésicule biliaire, occupant plus d'1/3 de son volume ; l'évolution de la maladie avec de fréquentes attaques de coliques biliaires, quelle que soit la taille des calculs; vésicule biliaire désactivée (non fonctionnelle); lithiase biliaire compliquée de cholécystite et/ou d'angiocholite; association avec une cholédocholithiase ; GSD compliqué par le développement du syndrome de Mirizzi ; cholélithiase, compliquée d'hydropisie, empyème de la vésicule biliaire; lithiase biliaire compliquée de perforation, pénétration, fistules; lithiase biliaire compliquée de pancréatite biliaire; GSD, accompagné d'une violation de la perméabilité du canal cholédoque et d'un ictère obstructif.

Avec la cholélithiase asymptomatique, ainsi qu'avec un seul épisode de coliques biliaires et des crises de douleur peu fréquentes, les tactiques d'attente sont les plus justifiées. Si indiqué, une lithotritie peut être pratiquée dans ces cas. Il n'est pas indiqué pour les porteurs de calculs asymptomatiques, car le risque de chirurgie l'emporte sur le risque de développer des symptômes ou des complications.

Dans certains cas, et uniquement sous des indications strictes, il est possible de réaliser une cholécystectomie laparoscopique en présence de porteurs de calculs asymptomatiques pour prévenir le développement de manifestations cliniques de lithiase biliaire ou de cancer de la vésicule biliaire. Indications de la cholécystectomie chez les porteurs asymptomatiques de calculs : vésicule biliaire calcifiée ("porcelaine") ; pierres de plus de 3 cm; le long séjour à venir dans la région avec le manque de soins médicaux qualifiés ; l'anémie falciforme; prochaine greffe d'organe au patient.

La cholécystectomie laparoscopique est moins traumatisante, a une période postopératoire plus courte, réduit la durée de séjour à l'hôpital et a un meilleur résultat esthétique. Dans tous les cas, il faut garder à l'esprit la possibilité de transférer l'opération à une opération ouverte en cas de tentatives infructueuses d'enlever la pierre par la méthode endoscopique. Il n'y a pratiquement aucune contre-indication absolue aux procédures laparoscopiques. Les contre-indications relatives incluent la cholécystite aiguë avec une durée de la maladie de plus de 48 heures, la péritonite, la cholangite aiguë, l'ictère obstructif, les fistules biliaires internes et externes, la cirrhose du foie, la coagulopathie, la pancréatite aiguë non résolue, la grossesse, l'obésité morbide, l'insuffisance cardiopulmonaire sévère.

La lithotripsie par ondes de choc est utilisée de manière très limitée, car elle a une gamme d'indications assez étroite, un certain nombre de contre-indications et de complications. La lithotritie extracorporelle par ondes de choc est utilisée dans les cas suivants : la présence dans la vésicule biliaire d'au plus trois calculs d'un diamètre total inférieur à 30 mm ; la présence de calculs qui « flottent » lors de la cholécystographie orale (signe caractéristique des calculs de cholestérol) ; une vésicule biliaire fonctionnelle, selon la cholécystographie orale ; réduction de la vésicule biliaire de 50%, selon la scintigraphie.

Il convient de garder à l'esprit que sans traitement supplémentaire à l'acide ursodésoxycholique, la fréquence de récurrence de la formation de calculs atteint 50%. De plus, la méthode n'empêche pas la possibilité de développer un cancer de la vésicule biliaire à l'avenir.

La papillosphinctérotomie endoscopique est indiquée principalement pour la cholédocholithiase.

Tous les patients atteints de lithiase biliaire sont soumis à l'observation du dispensaire en ambulatoire. Il est particulièrement nécessaire d'observer attentivement les patients porteurs de calculs asymptomatiques, de donner une évaluation clinique de l'anamnèse et des signes physiques. Lorsqu'une dynamique apparaît, examen de laboratoire et échographie. Des mesures similaires sont prises en cas d'antécédents d'un seul épisode de colique biliaire.

Lors de la conduite d'un traitement litholytique oral, une surveillance régulière de l'état des calculs par ultrasons est nécessaire. En cas de traitement par l'acide chénodésoxycholique, il est recommandé de surveiller les tests de la fonction hépatique une fois toutes les 2 à 4 semaines.

Dans un but de prévention, il est nécessaire de maintenir un indice de masse corporelle optimal et un niveau d'activité physique suffisant. Un mode de vie sédentaire contribue à la formation de calculs biliaires. Si la probabilité d'une diminution rapide du poids corporel du patient (plus de 2 kg / semaine pendant 4 semaines ou plus) est supposée, il est possible de prescrire des préparations d'acide ursodésoxycholique à une dose de 8-10 mg / (kg jour) pour empêcher la formation de pierres. Un tel événement empêche non seulement la formation proprement dite de calculs, mais également la cristallisation du cholestérol et une augmentation de l'indice de lithogénicité biliaire.

Les calculs biliaires peuvent être suspectés sur la base de manifestations cliniques, cependant, un diagnostic précis de la maladie des calculs biliaires n'est possible qu'après des études de laboratoire et radiologiques.

Le diagnostic différentiel de la lithiase biliaire, du syndrome douloureux de la lithiase biliaire et d'autres maladies peut nécessiter une radiographie non seulement de la cavité abdominale, mais également des organes thoraciques. Le caractère informatif de l'examen radiographique dans le diagnostic de lithiase biliaire est faible, puisque seulement 10% des calculs sont radio-opaques. Parfois, avec une occlusion de l'intestin grêle, de l'air se trouve dans l'arbre biliaire, ce qui indique la formation d'une fistule cholécysto-intestinale et d'une obstruction biliaire de l'intestin.

Test sanguin de laboratoire

Des études de la fonction hépatique (tests hépatiques) chez les patients suspects de cholécystolithiase doivent être réalisées sans faute. Dans certains cas, cependant, il n'y a pas de changement dans la lithiase biliaire. Une augmentation isolée de la concentration de bilirubine indirecte dans le sang est détectée avec un ictère suprahépatique, par exemple, en raison d'une hémolyse accrue. Les marqueurs biochimiques de l'ictère intrahépatique (par exemple, associé à l'hépatite) comprennent une augmentation de la concentration de bilirubine (directe et indirecte) et de l'activité des transaminases (AST et ALT) avec une activité de la phosphatase alcaline relativement normale ou légèrement élevée. Avec l'ictère sous-hépatique (mécanique), on observe une augmentation de la teneur en bilirubine directe dans le sérum sanguin et une augmentation de l'activité de la phosphatase alcaline avec une activité normale de l'AST et de l'ALT. Avec une cholangite à long terme, ainsi qu'avec une cholangite aiguë, l'activité des transaminases hépatiques dans le sérum augmente, ce qui est un signe de la destruction des hépatocytes. Dans la lithiase biliaire non obstructive, les écarts d'activité des transaminases par rapport au niveau normal sont minimes. Ces symptômes peuvent inciter à une cholangiographie peropératoire.

Environ 60% des patients atteints de lithiase biliaire (y compris les formes asymptomatiques) dans le diagnostic de lithiase biliaire montrent une modification du test sanguin biochimique. Aucune relation inverse n'a été trouvée dans ce cas; chez un grand nombre de patients présentant des écarts identifiés dans l'activité des enzymes hépatiques par rapport à la norme, le canal cholédoque ne contient pas de calculs. La bilirubine, la phosphatase alcaline et la g-glutamyl transpeptidase sont les marqueurs les plus sensibles de la lithiase biliaire parmi les tests couramment utilisés. Dans les cas aigus, il est nécessaire de déterminer l'activité de l'amylase ou de la lipase dans le sérum sanguin (pour exclure la pancréatite), ainsi que le nombre de leucocytes (une augmentation du taux peut indiquer une cholécystite aiguë).

Diagnostic échographique de la maladie des calculs biliaires

L'échographie () pour confirmer le diagnostic de la maladie des calculs biliaires est la plus largement utilisée. L'échographie est abordable et méthode sûre le diagnostic de la lithiase biliaire, qui ne cause presque aucune gêne au patient et peut être utilisé pour déterminer et évaluer les structures anatomiques de la cavité abdominale supérieure. À l'aide de l'échographie, vous pouvez évaluer l'état de la paroi de la vésicule biliaire et son contenu. Les calculs à l'échographie ressemblent à des formations hyperéchogènes prononcées à l'intérieur de la vésicule biliaire ; les grosses pierres créent une ombre acoustique qui descend de la pierre. Les calculs du canal cholédoque sont plus difficiles à voir. L'expansion du conduit en présence de petites pierres de la vésicule biliaire indique une cholédocholithiase. La fiabilité de l'échographie dans le diagnostic des calculs vésiculaires est très élevée. Avec une étude de haute qualité, des calculs peuvent être détectés chez au moins 95% des patients. La fiabilité de l'échographie dans la détection des calculs dans le canal cholédoque varie de 23 à 80 %, selon le physique du patient et l'expérience du diagnosticien.

Échographie endoscopique

La sensibilité de l'échographie endoscopique pour la détection des calculs du cholédoque est de 93 % et la spécificité de 97 %. La valeur diagnostique de la méthode se rapproche de celle de la rétrograde endoscopique (ERCP) - 89 et 100%, respectivement. Dans des études récentes, l'échographie endoscopique a été utilisée comme "l'étalon-or" pour diagnostiquer les calculs du canal cholédoque. L'étude a été réalisée sous anesthésie, lorsque des calculs ont été trouvés, une CPRE et une sphinctérotomie ont été réalisées. Il a été démontré que la sensibilité de l'échographie endoscopique est supérieure à celle de l'échographie transabdominale. SA Norton et D. Alderson ont prouvé la présence de lithiase biliaire chez 15 des 44 patients atteints de pancréatite idiopathique qui ont subi une échographie endoscopique.

Cholécystographie orale

La cholécystographie orale pour le diagnostic de la maladie des calculs biliaires perd de sa pertinence en raison de l'utilisation généralisée de l'échographie. La méthode est utilisée pour étudier les fonctions de la vésicule biliaire au fur et à mesure que l'agent de contraste s'y accumule. Probabilité d'obtenir résultat faux négatif lors du diagnostic de petits calculs, il est de 6 à 8%. Une diminution de la capacité fonctionnelle de la vessie est souvent observée dans la lithiase biliaire. Bien qu'il puisse entraîner l'apparition de pierres, il n'est pas considéré comme un facteur significatif. La cholécystographie orale peut être utilisée pour diagnostiquer la dyskinésie biliaire, mais n'est pas adaptée à la détection des calculs du cholédoque.

La tomodensitométrie dans le diagnostic de la maladie des calculs biliaires

La tomodensitométrie (TDM) dans le diagnostic de la maladie des calculs biliaires peut être une méthode plus fiable de détection des calculs dans le canal cholédoque que l'échographie. La spécificité de l'étude pour les calculs canalaires à l'origine d'un ictère obstructif est de 75 %. L'incidence relativement faible des calculs de la vésicule biliaire sur les tomodensitogrammes est due en partie à la présence de calculs de cholestérol, dont la densité ne diffère pas de celle de la bile au scanner. L'utilisation d'une nouvelle génération de tomographes hélicoïdaux et d'IRM augmente l'efficacité du diagnostic, cependant, les avantages potentiels de ces méthodes par rapport aux ultrasons ne sont pas évidents. Le scanner spiralé réalisé après une cholangiographie intraveineuse permet une évaluation fiable des changements structure anatomique vésicule biliaire et cholédoque.

balayage radio-isotopique

Acide hydroxy-imino-acétoacétique (HIDA - de l'acide hydroxy-imino-diacétique), marqué au technétium, après administration intraveineuse apparaît dans la bile. Ce type de contraste est utilisé pour étudier la perméabilité des voies biliaires ou pour détecter des anastomoses biliodigestives. Les capacités de la méthode à détecter les calculs de la vésicule biliaire sont limitées, bien qu'une mauvaise visualisation de la vésicule biliaire à la suite d'un blocage des voies biliaires aide à diagnostiquer une cholécystite aiguë. L'analyse radioisotopique avec HIDA pour le diagnostic de la maladie des calculs biliaires est raisonnable chez les patients souffrant de douleur, de fièvre, de cholécystolithiase et de pneumonie du lobe inférieur droit. Le syndrome douloureux et la faiblesse susmentionnés peuvent survenir lorsque diverses maladies, tandis que la détection d'une vésicule biliaire fonctionnelle permet d'exclure une cholécystite. Le balayage radio-isotopique avec HIDA dans l'ictère obstructif n'a aucune valeur, car l'isotope ne peut pas pénétrer dans les voies biliaires pendant l'obstruction.

Cholangiographie intraveineuse dans le diagnostic de la maladie des calculs biliaires

L'introduction a conduit au fait que la cholangiographie intraveineuse a de nouveau été utilisée pour diagnostiquer les calculs dans le canal cholédoque. Cependant, dans 3 à 10 % des cas, les conduits ne peuvent pas être suffisamment contrastés. Malgré l'amélioration de la méthode (dont la possibilité de son utilisation pour la visualisation des voies biliaires comme alternative à l'échographie et le faible risque d'allergisation), le temps de l'étude, le coût et, dans certains cas, l'inefficacité la rendent moins attrayant. De plus, l'utilisation de la cholangiographie intraveineuse est limitée chez les patients prenant des médicaments contenant de l'iode, ainsi que chez les patients présentant une obstruction. voie biliaire, car dans le dernier cas agent de contraste n'entre pas dans les conduits.

Cholangiographie par résonance magnétique

Développement de nouvelles méthodes de visualisation rapide et d'amélioration Logiciel a permis de comparer le contenu informatif de la cholangiographie par résonance magnétique (MRCH) avec la CPRE. La méthode est basée sur l'affichage d'un milieu liquide stationnaire, ce qui permet une étude détaillée de la bile et du liquide dans l'estomac, le duodénum. Les images sont meilleures avec des voies biliaires dilatées et le flux biliaire peut être confondu avec des calculs. La sensibilité de la méthode dans le diagnostic de la maladie des calculs biliaires et la détection des calculs dans le canal cholédoque dépasse 95 %, la spécificité est de 89 % et la précision est de 92 %. La possibilité d'identifier les caractéristiques anatomiques des voies biliaires extrahépatiques n'a pas été prouvée. Après avoir mené des études non invasives standard, T.H. Liu a divisé les patients suspects de calculs du cholédoque en quatre catégories. Les patients extrêmement risque élevé cholédocholithiase, la CPRE a été réalisée. Les patients à risque de cholédocholithiase ont subi une MRCP suivie d'une CPRE (si des calculs ont été trouvés). Il a été démontré qu'avec une précision de diagnostic MRCP de plus de 90 %, dans de nombreux cas, la CPRE n'est pas nécessaire. L'efficacité de la MRCG dépend de la conduite correcte et de la capacité à interpréter les résultats. Le capital et les coûts permanents de l'IRM limitent encore son adoption généralisée.

Cholangiographie transhépatique percutanée

La cholangiographie transhépatique percutanée est indiquée pour les voies biliaires dilatées. Cette méthode ne doit pas être utilisée systématiquement si une obstruction du conduit par des calculs est suspectée. Malgré l'utilisation d'aiguilles de ponction fines 5G, il existe un risque de bile et de saignement chez les patients présentant des troubles thrombotiques.

Cholangiopancréatographie rétrograde endoscopique dans le diagnostic des calculs biliaires

L'ERCP est la « norme de référence » pour l'imagerie préopératoire du canal cholédoque dans le diagnostic de la maladie des calculs biliaires. Après avoir examiné l'OBD à l'aide d'un duodénoscope (endoscope à optique latérale), il est canulé séparément pour identifier le canal cholédoque et le canal pancréatique. Le produit de contraste hydrosoluble est injecté directement dans les voies biliaires. Cela donne à la CPRE un avantage sur les autres méthodes de visualisation des voies biliaires grâce à la possibilité d'effectuer des manipulations thérapeutiques - sphinctérotomie et lithoextraction.

L'article a été préparé et édité par : chirurgien

MINISTERE DE LA SANTE DE LA FEDERATION DE RUSSIE

UNIVERSITÉ MÉDICALE D'ÉTAT

DÉPARTEMENT DES MALADIES CHIRURGICALES №2

Chef de département : Professeur

Malade:Diagnostic:Lithiase biliaire, cholécystite aiguë

Conservateur-étudiant :4èmecours7 semestre groupes

Chef d'équipe

curation :deavant deN° d'antécédents : Note de l'enseignant sur le test :

INFORMATIONS GÉNÉRALES SUR LE PATIENT

Nom complet du patient :

Âge: 48 ans

Adresse:

Sol: Femelle

Lieu de travail:

Date de réception:

Méthode d'entrée : urgence

Date de conservation :

Diagnostic: Lithiase biliaire, cholécystite aiguë

Opération: Cholécystectomie laparoscopique. Drainage

cavité abdominale (14/10/2005 à 9:40)

Anesthésie: endotrachéal

Groupe sanguin: A(II) Rh+

Issue de la maladie : amélioration

Employabilité : restauration en cours

PLAINTES DU PATIENT

(Au moment de la conservation)

Le patient se plaint de fortes douleurs dans l'hypochondre droit. Selon le patient, la douleur survient spontanément, sans raison apparente (rarement après avoir mangé). La durée de la douleur est de 0,5 à 2 heures. De plus, le patient se plaint également de bouche sèche, de nausées, de vomissements, de faiblesse générale.

ANTÉCÉDENTS DE LA MALADIE (Anamnèsemorbide)

Il se considère malade depuis 14 heures, lorsque des douleurs intenses ont commencé à apparaître dans l'hypochondre droit. Les douleurs disparaissaient parfois d'elles-mêmes et parfois après avoir mangé (j'ai arrêté le médicament mezim et j'ai pris Charbon actif). Je n'ai pas consulté de médecins. Était fort mal de tête, puis la sueur a commencé, et après cela, elle a commencé, vile, il y a eu une augmentation de la pression artérielle = 150/100, la température n'a pas dérangé. Il y avait aussi des nausées et des vomissements. Une détérioration de l'état a été notée au cours des 4 dernières heures. Irradiation de la douleur dans le cœur et dans la région lombaire droite.

ANAMNÈSE DE LA VIE(Anamnèsevitae)

Informations biographiques générales : est né dans l'année, dans une famille prospère, le sixième enfant d'affilée. Croissance et développement selon l'âge. Enfant, elle souffrait de rhumes. Elle était une excellente élève à l'école. Etat civil : marié avec 3 enfants. Conditions de vie: satisfaisant, vit dans un appartement de 3 pièces. Il y a 3 personnes dans la famille, la sécurité matérielle est satisfaisante. Repas réguliers pendant le travail. Mode temps libre : le sommeil n'est pas calme (perturbé), physique. la culture et le sport ne sont pas concernés. À ce moment: sommeil calme, sans maux de tête.

Condition de travail: Des maladies transférées : hron. pancréatite, appendicectomie (1981), cardiopathie ischémique, lithiase biliaire. Mauvaises habitudes : non. Diabète sucré, hépatite, SIDA nie

Antécédents familiaux héréditaires : pas alourdi.

Antécédents allergiques : allergique r-th nie.

DONNÉES DE L'ÉTUDE OBJECTIVE(statutpraesens)

INSPECTION GÉNÉRALE

Etat général du patient : satisfaisant

Conscience : claire

Poste : actif

Allure : droite

Posture : droite

Hauteur : 165 cm.

Poids : 80kg..

Indice Brocca : 80*100%/165-100=123.07% (physique fort)

Indice de Pinier : 165-(80+82)=3 (physique fort)

Indice de Quetelet : 80/(1,65)2=29,38 (physique fort)

Type de corps : hypersthénique

Température corporelle : 36,6

expression du visage : calme

Langue : humide, enduite de blanc

État des dents : satisfaisant

Sclérotique muqueuse de l'œil, du nez, de la bouche, de l'oreille : propre sans écoulement (sans caractéristiques) - satisfaisante.

Forme du visage : ovale, yeux non dilatés

Peau : Couleur rose pâle. Les zones de pigmentation, les éruptions cutanées, les varicosités, les hémorragies n'ont pas été détectées.

Cicatrices : Dans la région iliaque droite postopératoire (appendicectomie)

Type de cheveux : type féminin.

Turgescence cutanée : préservée, élastique.

Humidité de la peau : Sèche.

Ongles : forme régulière (couleur rose).

Graisse sous-cutanée : fortement développée.

Oedème : non.

Ganglions lymphatiques : non agrandis

Système musculaire : Indolore, degré de développement satisfaisant (l'action dynamique et statique est satisfaisante).

Système squelettique: Le rapport des os du squelette est proportionnel. La forme des os est correcte, sans épaississement, ni déformations. Il n'y a pas de douleur à la palpation. Le symptôme des "doigts de tambour" est absent.

Les articulations: forme régulière et tailles. Douleur en se penchant articulation du genou droite et gauche dans la région de l'épicondyle médial. Plein mouvement.

SYSTÈME RESPIRATOIRE

Inspection: Nez : De forme normale, muqueux clair sans écoulement.

Larynx : sans déformations ni tuméfaction.

Formulaire gr. classe : hypersthénique, symétrique.

Fosse supérieure et sous-clavière : modérément prononcée, identique des deux côtés.

Largeur des espaces intercostaux : 1,5 cm.

Angle épigastrique : aigu.

Côtes latérales : Modérément obliques.

L'ajustement des omoplates à Ch. classe : serré, et situé au même niveau classe d'excursion classe : 3cm.

Type de respiration : principalement abdominale, gr.cl. participe à l'acte de respirer uniformément. La respiration est effectuée en silence, sans la participation des muscles auxiliaires.

BH : 18 en 1 minute.

Respiration : profondeur moyenne, rythmée.

Le rapport des phases d'inspiration et d'expiration: non rompu.

Palpation: Angle épigastrique : aigu.

Côtes : intactes.

Résistance (élasticité) : Élastique, élastique, souple.

Douleur: lors de la sensation des côtes, des espaces intercostaux, les muscles pectoraux n'ont pas été détectés.

Percussion: Comparatif : un son pulmonaire clair est déterminé sur toute la surface des poumons.

Topographique :

ligne topographique

Poumon droit

Poumon gauche

Limite supérieure

Hauteur des plateaux devant

3 cm au-dessus de la clavicule

3 cm au-dessus de la clavicule

Hauteur debout des plateaux à l'arrière

7 vertèbre cervicale

7 vertèbre cervicale

En bout de ligne

Le long de la ligne péritonéale

Bord supérieur 6 côtes

non défini

Le long de la ligne médio-claviculaire

non défini

Le long de la ligne axillaire antérieure

Le long de la ligne médio-axillaire

Le long de la ligne axillaire postérieure

Le long de la ligne des épaules

Le long de la ligne vertébrale

Processus épineux 11 coffres. vertèbre

Largeur des champs d'un krening : 4 cm. des deux côtés.

Mobilité respiratoire du bord inférieur des poumons :

Ligne médio-axillaire : 7cm droite et gauche

Sur les lignes médio-claviculaires : à droite 5 cm à gauche n'est pas déterminé

Sur les lignes scapulaires : 5 cm. droite et gauche

Auscultation: la respiration visiculaire est déterminée au-dessus du poumon des deux côtés. Les bruits respiratoires indésirables et le frottement pleural ne sont pas entendus. La bronchophonie n'est pas modifiée des deux côtés.

système circulatoire

Inspection: Pulsation améliorée artères carotides(danse de la carotide), gonflement des veines jugulaires, aucune pulsation visible des veines n'a été trouvée.

La saillie de la région du cœur, les pulsations visibles (impulsion apicale et cardiaque, pulsation épigastrique) ne sont pas déterminées visuellement.

Palpation: Le battement de l'apex est situé dans l'espace intercostal en V à 1 cm en dedans de la ligne médio-claviculaire gauche, largeur 1 cm, force faible et modérée. Le phénomène de tremblement diastolique et systolique dans la région péricardique, la pulsation épigastrique n'est pas palpable. À la palpation dans la région du cœur, la douleur n'est pas déterminée.

Percussion:

Stupidité relative

Stupidité absolue

4ème espace intercostal 0,5 - 1 cm à droite du bord du sternum

4ème espace intercostal sur le côté gauche du sternum

5ème espace intercostal 1 cm en dedans de la ligne médio-claviculaire gauche

De la zone de l'impulsion apicale se déplacer vers le centre (1,5 cm médialement)

Ligne parasternale 3ème espace intercostal

4 espace intercostal

La Coupe transversale émoussé : 12cm.

La largeur du faisceau vasculaire est de 6 cm, 2ème espace intercostal à gauche et à droite.

Configuration cardiaque : normale. Ascultation : Tonalités : - battements de cœur rythmés

Le nombre de battements de coeur - 76

Premier ton de sonorité normale

Deuxième ton de sonorité normale

Les tonalités supplémentaires ne sont pas entendues

Murmures : inaudibles, le frottement péricardique n'est pas audible.

Pouls artériel sur les artères radiales : symétrique, élastique, fréquence = 76 battements. En 1 min., rythmique, tension modérée, ample.

Pression artérielle sur les artères brachiales : 120/70 mm. rt. Art.

SYSTÈME DIGESTIF

Examen de la cavité buccale.

Il n'y a pas d'odeur; muqueuse de la surface interne des lèvres, des joues, douce et palais dur couleur normale ; les éruptions cutanées, les ulcérations sont absentes; les gencives ne saignent pas; langue de taille et de forme normales, humide, sans poils ; les papilles filiformes et fongiformes sont bien exprimées; pharynx de couleur normale ; les arcs palatins sont bien profilés ; les amygdales ne dépassent pas les arcs palatins; la membrane muqueuse du pharynx n'est pas hyperémique, humide, la surface est lisse. Sclère de couleur normale.

Formule dentaire :

8 7 6 5 4 3 2 1 1 2 3 4 5 6 7 8

8 7 6 5 4 3 2 1 1 2 3 4 5 6 7 8

Légende:6 - dent cariée, 6 - dent extraite, 6 - dent scellée, 6 - dent prothétique.

Examen de l'abdomen.

L'abdomen est de forme normale, symétrique, les collatérales sur la face antérieure de l'abdomen et ses faces latérales ne sont pas exprimées ; il n'y a pas de péristaltisme pathologique; les muscles de la paroi abdominale sont impliqués dans l'acte de respirer. Lorsqu'elles sont vues en position verticale, les saillies herniaires n'ont pas été trouvées. En réponse à la toux, il n'y a pas d'augmentation de la douleur dans l'épigastre.

Circonférence abdominale 90 cm.

Percussion approximative de l'abdomen.

Une tympanite de gravité variable est déterminée, la matité dans les zones en pente de l'abdomen n'est pas observée.

Palpation superficielle approximative de l'abdomen.

Une douleur modérée dans la région de l'hypochondre droit est déterminée, le symptôme de Shchetkin-Blumberg est négatif. Lors de l'examen des «points faibles» de la paroi abdominale antérieure (anneau ombilical, aponévrose de la ligne blanche de l'abdomen, anneaux inguinaux), aucune saillie herniaire ne se forme.

Avec palpation glissante méthodique profonde de l'abdomen selon la méthode Obraztsov-Strajesko-Vasilenko:

Le côlon sigmoïde est palpable dans la région inguinale gauche à la limite des tiers médian et externe de la linea umbilicoiliaceae sinistra sur 15 cm, de forme cylindrique, de diamètre - 2 cm, de consistance densément élastique, à surface lisse, mobilité dans les 3- 4 cm, indolore, ne ronronne pas ; le caecum est palpé dans la région inguinale droite à la limite des tiers médian et externe de linea umbilicoiliaceae dextra sous la forme d'un cylindre avec une expansion en forme de poire vers le bas, consistance molle-élastique, 3-4 cm de diamètre, légèrement grondant à la palpation, les parties restantes de l'intestin n'ont pas pu être palpées.

Palpation de l'estomac et détermination de son bord inférieur :

Par percussion, par palpation stétoacoustique, le bord inférieur de l'estomac est déterminé à 3 cm au-dessus du nombril.

Il n'a pas été possible de déterminer le bord inférieur de l'estomac par palpation profonde de la grande courbure de l'estomac, par la méthode du bruit d'éclaboussure.

La petite courbure et le pylore ne sont pas palpables ; le bruit d'éclaboussure à droite de la ligne médiane de l'abdomen (symptôme de Vasilenko) n'est pas déterminé.

Auscultation de l'abdomen.

L'auscultation abdominale révèle des bruits intestinaux péristaltiques normaux.

Détermination des limites de la matité hépatique absolue.

La méthode de percussion détermine :

borne supérieure

sur la ligne axillaire antérieure droite - 7e côte

sur la ligne médio-claviculaire droite - 6ème côte

sur la ligne parasternale droite - 5ème côte

ligne de fond

sur la ligne axillaire antérieure droite - à 2 cm de la 10e côte. sur la ligne médio-claviculaire droite - 4 cm vers le bas à partir de l'arc costal

sur la ligne parasternale droite - 5 cm vers le bas à partir de l'arc costal

le long de la ligne médiane antérieure - 9 cm vers le bas à partir de la base du processus xiphoïde bordure gauche la matité hépatique dépasse la ligne péristernale le long du bord de l'arc costal de 2 cm.

dimensions de la matité hépatique :

sur la ligne axillaire antérieure droite - 15 cm,

sur la ligne médio-claviculaire droite - 15 cm

le long de la ligne péristernale - 13 cm

le long de la ligne médiane antérieure - 13 cm

taille oblique (selon Kurlov) - 9 cm

le foie est palpé à 1 cm sous le bord de l'arc costal (le long de la ligne médio-claviculaire droite) ; le bord du foie est dense, uniforme, avec une surface lisse, légèrement pointue; le foie est indolore; la vésicule biliaire n'est pas palpable; douleur à la palpation au point de projection de la vésicule biliaire, les symptômes d'Ortner, Zakharyin, Vasilenko, Murphy, Georgievsky-Mussi sont faiblement positifs.

Percussion de la rate.

Le long de la ligne passant 4 cm en arrière et parallèle à la ligne costo-articulaire gauche, les limites de la matité splénique ont été déterminées :

supérieur - au niveau de la 9ème côte;

celle du bas est au niveau de la 11e côte.

le bord antérieur de la matité splénique ne dépasse pas la linea costoarticularis sinistra.

dimensions de la matité splénique : diamètre - 6 cm ; longueur - 8 cm.

La rate n'est pas palpable.

Examen du pancréas.

Le pancréas n'est pas palpé ; la douleur à la palpation dans la zone de Chauffard et le point pancréatique de Desjardin est absente ; Le symptôme de Mayo-Robson est négatif.

Examen du rectum.

Lors de l'examen de l'anus, la macération, l'hyperémie de la peau de la région périanale est absente. A l'examen digital : tonicité sphinctérienne normale, absence de tumeurs, infiltrats inflammatoires, hémorroïdes compactées. Prostate forme, consistance et taille normales, indolore.

SYSTÈME URINAIRE

Inspection: Région lombaire : hyperhémie cutanée, gonflement,

il n'y a pas de lissage des contours.

Région sus-pubienne : un renflement limité n'a pas été trouvé.

Percussion: Région lombaire : le symptôme de Pasternatsky est négatif.

Palpation: Les reins en décubitus dorsal et debout ne sont pas palpables.

La vessie est indolore, élastique, palpée sous la forme d'une formation sphérique élastique douce au-dessus de l'articulation pubienne. La palpation pénétrante des reins et des uretères des deux côtés est indolore (points vertébraux urétéraux et costaux).

SYSTÈME GÉNÉRAL.

Inspection: Type de cheveux selon le type féminin. La voix est basse. Glandes mammaires sans modifications pathologiques (pas d'œdème, d'hyperémie, de rétractions...).

SYSTÈME ENDOCRINIEN.

Inspection et palpation : Le développement physique et mental correspond à l'âge. Les caractères sexuels secondaires correspondent au sexe. Les tremblements des paupières, de la langue et des doigts ne sont pas définis.

La forme du cou est normale, les contours sont réguliers, indolores à la palpation. La glande thyroïde n'est pas hypertrophiée, indolore, mobile. L'obésité est négligeable.

SYSTÈME NERVEUX ET CAPTEURS

Inspection: La mémoire, l'attention, le sommeil sont sauvegardés. Humeur - gaie, apte. Restriction de l'activité motrice : non. Il n'y a pas d'écarts dans la zone sensible.

L'état de la psyché - conscience est clair, normalement orienté dans l'espace, le temps et la situation.

L'intelligence correspond au niveau de développement.

Le comportement est approprié.

Équilibré, sociable.

Aucun écart n'est constaté

Sphère motrice : Démarche stable,

sans douleur.

Les crampes et les contractures musculaires ne sont pas déterminées.

Réflexes : cornéens, pharyngés, tendineux

enregistré. Pathologique (Babinsky et

Rossolimo) sont absents.

L'exophtalmie et l'énophtalmie sont absentes.

DIAGNOSTIC PRÉLIMINAIRE

JUSTIFICATION DU DIAGNOSTIC

Selon les données suivantes, le patient peut être suspecté d'avoir une lithiase biliaire : le patient se plaint de douleurs dans l'épigastre et l'hypochondre droit, d'intensité modérée, d'une durée de 0,5 à 2 heures, de nausées, de vomissements, d'une faiblesse générale, le patient se considère environ 14 ans heures.

Examen objectif : douleur au point de projection Vessie, manque de tension protectrice des muscles abdominaux Symptôme de Carey, Murphy, Ortner faiblement positif

PLAN D'ENQUÊTE

3. Réaction de Wasserman

4. ECG en 12 dérivations

5. Radiographie pulmonaire

6. Étude du contenu gastrique par méthode fractionnée

7. Sondage duodénal

8. Analyse des matières fécales (scatologique, pour dysbactériose, protozoaires, bactérie.)

9. Diastasis urinaire, amylase sanguine

10. Renoscopie de l'estomac et des intestins avec radiographies ciblées

11. Irrigoscopie

12. Oesophagoscopie

13. Gastroduodénoscopie avec biopsie

14. Sigmoïdoscopie, coloscopie

15. Biochimie du sang

16. Test d'urine pour l'urobiline et la bilirubine

17. Examen des matières fécales pour la stercobiline

18. Cholécystographie

19. Analyse radio-isotopique et échographique du foie et de la rate, du pancréas

20. Détermination de la classe des immunoglobulines sanguines

21. Test sanguin pour l'antigène HbS

22. CT scan des organes abdominaux

23. L'étude des réactions sérologiques (RW, anticorps anti-VIH, marqueurs de l'hépatite virale, dont les anticorps anti-antigène HBs)

24. Détermination du groupe sanguin, facteur Rh

Analyse générale du sang

Indicateurs

Données

135g/l

123g/l

Leucocytes (9.10.2005)

Érythrocytes (10.10.2005)

9,0 10 9 /l

8,8 10 9 /l

8,0 10 9 /l

6,3 10 9 /l

4.2 10 12 /l

3,8 10 12 /l

Myélocytes

métamyélocytes

Neutrophiles poignardés (12.10.2005)

Neutrophiles segmentés (12.10.2005)

Éosinophiles (12.10.2005)

Basophiles

Lymphocytes (12.10.2005)

Monocytes

Cellules plasmatiques

19 mm/h

Analyse d'urine.

10.10.2005

Indicateurs

Données

Jaune

Transparence

modérément

Densité relative

neutre

Négatif

Épithélium :

Appartement

3-3-2 en vue

Leucocytes

2-1 en vue

Urat 11

Test sanguin biochimique

12.10.2005

Indicateurs

Données

protéines totales

70g/l

Créatinine

61 mmol/litre

bilirubine totale

10,8 µmol/l

DÉTERMINATION DU GROUPE SANGUIN 12.10.2005

PTI 12.10.2005

MICROREACTION AVEC L'ANTIGENE CARDIOLIPINE 12.10.2005

NÉGATIF

ECG 10.10.2005

Conclusion : Rythme sinusal avec fréquence cardiaque = 75, EOS dévié vers la gauche, symptômes d'hypertrophie myocardique ventriculaire gauche avec signes de dystrophies.

SPIROGRAPHIE 13.10.2005

Conclusion : - La violation de la perméabilité bronchique n'a pas été détectée, sans signe de rétrécissement expiratoire des voies respiratoires.

VC dans la plage normale.

L'état de l'appareil de ventilation est dans les limites normales.

FIBRO-OESOPHAGOGASTRODOUDENOSCOPIE 11.10.2005

L'œsophage est librement praticable. L'emboîture du cardia se ferme complètement. Le test d'effort est négatif. Dans l'estomac, il y a une quantité modérée de bile jaunâtre transparente, les plis sont alambiqués, juteux. Perilstaltics peut être tracé dans tous les départements. La muqueuse gastrique est hyperémique, modérément amincie. Le portier de forme arrondie est passable, la membrane muqueuse du bulbe est de 12 p.k. parsemée de petites éruptions de couleur blanchâtre comme de la "semoule".

Conclusion : dyskinésie biliaire. Gastrite subtrophique superficielle. Image endoscopique de pancréatite.

ultrason Foie : visualisé en fragments à travers l'espace intercostal, augmentation de l'échogénicité.

Vésicule biliaire: Après avoir mangé, elle augmente à 100 * 36 mm., Le mur est épaissi jusqu'à 4 m.. Dans la cavité, les pierres mesurent jusqu'à 14 mm. L'un d'eux est fixé à la bouche. Le CBP n'est pas étendu.

Pancréas: augmentation de la surface corporelle jusqu'à 23 mm, augmentation de l'échogénicité, hétérogène.

Reins : La topographie et les dimensions sont normales. Scellement des déformations des parois du CLS. L'écoulement de l'urine n'est pas perturbé. Inclusions hyperchromiques simples bien marquées jusqu'à 1-3 mm.

Rate : non hypertrophiée.

Conclusion : les signes d'écho de la cholécystite calculeuse aiguë ne sont-ils pas exclus ? Pierre à la bouche.

DIAGNOSTIC CLINIQUE

Lithiase biliaire. Cholécystite aiguë.

JUSTIFICATION DU DIAGNOSTIC

Selon les données suivantes, le patient peut être suspecté d'avoir une lithiase biliaire : le patient se plaint de douleurs dans l'épigastre et l'hypochondre droit, d'intensité modérée, d'une durée de 0,5 à 2 heures, de nausées, de vomissements, d'une faiblesse générale, le patient se considère environ 14 ans heures.

Pour confirmer le diagnostic, les études suivantes ont été réalisées:

Examen objectif : douleur au point de projection de la vessie, absence de tension protectrice des muscles abdominaux. Symptôme Carey, Murphy, Ortner faiblement positif

Les méthodes de recherche instrumentale prouvent également l'exactitude du diagnostic:

La conclusion de l'échographiste: il y a des signes de cholécystite calculeuse aiguë, la vésicule biliaire est agrandie jusqu'à 100 * 36 mm, la paroi est épaissie jusqu'à 4 mm, la présence de calculs dans la cavité jusqu'à 14 mm . et à l'embouchure de la vésicule biliaire. Le CBP n'est pas étendu.

DIAGNOSTIC DIFFÉRENTIEL

La lithiase biliaire doit être différenciée des maladies suivantes : pancréatite chronique, gastrite chronique, duodénite, ulcère gastrique et douze l'ulcère duodénal, cholécystite alithiasique, tumeur de la moitié droite du côlon, cancer de la vésicule biliaire. Étant donné que les symptômes et cours clinique des maladies ci-dessus sont similaires et une erreur de diagnostic peut conduire à complications graves en raison de tactiques de traitement mal choisies. Considérez séparément les différences entre chacune des maladies ci-dessus de la lithiase biliaire:

ulcère peptique duodénum: Cette maladie se caractérise par la présence de périodes d'exacerbation et de rémission, et il n'y a pas d'évolution dynamique de la lithiase biliaire.

Douleur à ulcère peptique les ulcères duodénaux sont diurnes et rythmiques (faim, douleurs nocturnes), lors d'une exacerbation, des douleurs prolongées durant 3-4 semaines sont caractéristiques. La lithiase biliaire se caractérise par des douleurs liées à la prise d'aliments gras et "lourds", la douleur est soulagée par la prise d'antispasmodiques, et est de courte durée. La douleur est généralement localisée au point de projection de la vésicule biliaire, les symptômes d'Ortner, Georgievsky - Mussi sont positifs.

Avec l'ulcère duodénal, la température corporelle reste généralement normale et avec la lithiase biliaire, elle est souvent de nature subfébrile.

Les paramètres sanguins de l'ulcère duodénal sont les suivants: la VS est normale, augmente avec les complications, l'image du sang blanc est normale, avec des complications de saignement, une anémie est observée. Dans la lithiase biliaire, la RSE augmente, une hyperleucocytose est observée avec un déplacement vers la gauche.

Les vomissements après avoir mangé 2 à 2,5 heures après avoir mangé, ce qui apporte un soulagement, est caractéristique de l'ulcère duodénal, et avec la lithiase biliaire, les vomissements n'apportent pas de soulagement, ils contiennent un mélange de bile. La fonction sécrétoire de l'estomac, en règle générale, reste normale, et avec l'ulcère duodénal, un état hyperacide est généralement observé.

Les saignements dans l'ulcère duodénal ont généralement des manifestations caractéristiques: vomissements de la forme " Marc de café", méléna, blanchiment de la peau et lithiase biliaire ne se produisent pas.

Les données d'œsophagogastroduodénoscopie avec examen histologique d'échantillons de biopsie sélectionnés et examen radiologique de l'estomac permettent un diagnostic plus précis. Ce patient ne dispose pas de données œsophagogastroduodénoscopiques en faveur d'un ulcère peptique. L'examen de la muqueuse gastrique a révélé une gastrite atrophique et l'examen des papilles faterii n'a révélé aucune fuite de bile.

Diagnostic différentiel entre ulcère gastrique et lithiase biliaire :

Avec l'ulcère gastrique, la douleur survient immédiatement après avoir mangé ou 15 à 45 minutes après avoir mangé. Le soulagement dans cette condition peut amener l'évacuation du contenu gastrique. Dans la lithiase biliaire, la douleur est généralement associée à la consommation d'aliments gras, frits, épicés et les vomissements n'apportent pas de soulagement, ils contiennent des impuretés biliaires.

Localisation de la douleur dans l'ulcère peptique, en règle générale, entre le processus xiphoïde et le nombril, le plus souvent à gauche de la ligne médiane, irradiation vers la moitié gauche de la poitrine, vers la région interscapulaire. Avec la lithiase biliaire, la douleur est localisée dans l'hypochondre droit, irradiant vers l'omoplate droite, l'épaule droite. La douleur est située à un point caractéristique - le point de projection de la vésicule biliaire, les symptômes d'Ortner, Georgievsky-Mussi sont également positifs.

L'acidité du suc gastrique avec l'ulcère peptique change et avec la lithiase biliaire, elle est normale.

Le patient n'a pas les caractéristiques pour l'ulcère peptique de l'estomac et du duodénum.

Diagnostic différentiel entre cholécystite chronique alithiasique et lithiase biliaire :

Le tableau clinique de la cholécystite chronique alithiasique est similaire à celui de la cholécystite chronique calculeuse, cependant, la douleur dans l'hypochondre droit n'est pas si intense, mais diffère par sa durée, son caractère presque constant, son intensification après des troubles alimentaires (aliments gras, frits, en particulier en cas d'excès les montants). Pour diagnostiquer le plus méthodes informatives sommes procédure d'échographie et la cholécystocholangiographie.

La douleur du patient est intermittente, d'intensité modérée. Les données de diagnostic échographique confirment la présence de structures d'échogénicité accrue.

Diagnostic différentiel entre tumeur de la vésicule biliaire et lithiase biliaire.

Dans une tumeur de la vésicule biliaire et des voies biliaires, des signes de généralisation du processus cancéreux sont révélés : signes généraux, tels que faiblesse, fatigue, manque d'appétit, perte de poids, anémie ; combiné avec des symptômes locaux - un foie tubéreux hypertrophié, une ascite et une jaunisse.

Chez ce patient, la présence d'un processus tumoral est rejetée par les données échographiques.

Diagnostic différentiel entre appendicite aiguë et lithiase biliaire.

La nature de la douleur : à l'épigastre, d'intensité modérée (traction constante), puis se déplace vers la région iliaque droite. Autres plaintes : nausées, vomissements, rétention de selles, fièvre. Développement : aigu. Examen objectif: douleur et tension musculaire dans la région iliaque droite, irritation du péritoine, Voskresensky, Rozdolsky, Obraztsov, Ravzing, Sitkovsky, intoxication prononcée. Examens complémentaires: une modification du volume sanguin total indiquant une inflammation.

Diagnostic différentiel entre colique néphrétique et lithiase biliaire.

La nature de la douleur : dans le bas du dos (paroxystique), extrêmement intense avec irradiation au niveau de l'aine, atténuée par l'utilisation d'antispasmodiques. Autres plaintes : dysurie possible. Anamnèse : lithiase urinaire. Développement : aigu. Examen objectif : la palpation de l'abdomen est généralement indolore, sm positif Pasternatsky, il n'y a pas d'intoxication. Examens complémentaires : érythrocyturie.

ETIOLOGIE ET ​​PATHOGENESE

Le substrat morphologique de la lithiase biliaire est le calcul de la vésicule biliaire des voies biliaires. Les calculs biliaires sont constitués des composants habituels de la bile - la bilirubine, le cholestérol et le calcium. Le plus souvent, il y a des calculs mixtes, cependant, avec la prédominance d'un composant, ils parlent de cholestérol, de pigments ou de calculs calcaires.

Il existe trois raisons principales à leur formation: une violation de l'équilibre physico-chimique de la composition de la bile, des modifications inflammatoires de l'épithélium de la vésicule biliaire et une stagnation de la bile.

Violation de l'équilibre physico-chimique de la composition de la bile.

Avec la lithiase biliaire, il y a un changement dans la teneur normale en cholestérol, en lécithine et en sels biliaires dans la bile. Les structures micellaires, constituées d'acides biliaires et de lécithine, contribuent à la dissolution du cholestérol dans la bile, qui fait partie des micelles. Dans les structures micellaires, il existe toujours une certaine marge de solubilité du cholestérol. Lorsque la quantité de cholestérol dans la bile dépasse ses limites de solubilité, la bile devient sursaturée en cholestérol et commence à cristalliser et à précipiter. La lithogénicité de la bile est caractérisée par l'indice de lithogénicité, qui est déterminé par le rapport de la quantité de cholestérol dans cette bile à la quantité de cholestérol pouvant être dissoute à un rapport donné d'acides biliaires, de lécithine et de cholestérol.

La bile devient lithogène avec les changements suivants dans le rapport de ses composants :

1) une augmentation de la concentration de cholestérol (hypercholestérolémie)

2) diminution de la concentration des phospholipides

3) diminution de la concentration des acides biliaires.

Les raisons conduisant à une diminution du flux d'acides biliaires dans la bile peuvent être divisées en trois groupes :

1) Diminution de la synthèse des acides biliaires et perturbation des mécanismes de rétroaction, ainsi que des mécanismes régulant la synthèse des acides biliaires : altération de la fonction hépatique, apport médicaments hormonaux(corticostéroïdes, contraceptifs oraux, etc.), grossesse, augmentation des taux d'hormones œstrogènes, intoxication par des poisons hépatotoxiques.

2) Violation de la circulation entérohépatique des acides biliaires (des pertes importantes d'acides biliaires se produisent lors de la résection de la partie distale intestin grêle, maladies de l'intestin grêle).

3) Drainage des acides biliaires de la vésicule biliaire, qui s'observe avec atonie de la vésicule biliaire, famine prolongée.

Stagnation de la bile

La violation du travail coordonné des sphincters des voies biliaires provoque diverses dyskinésies dans la nature. Attribuer une dyskinésie hypertonique et hypotonique des voies biliaires et de la vésicule biliaire.

Avec les formes hypertoniques de dyskinésie, il y a une augmentation du tonus des sphincters. Un spasme dans la partie commune du sphincter d'Oddi provoque une hypertension dans les conduits et dans la vésicule biliaire.

Dans les formes hypotoniques de dyskinésie, le sphincter d'Oddi se détend, suivi d'un reflux du contenu du duodénum dans les voies biliaires, et une infection des voies se produit. Avec la dyskinésie, il y a une violation de l'évacuation de la bile de la vésicule biliaire et des conduits, ce qui est un facteur prédisposant à la formation de calculs.

Infections des voies biliaires

L'état de la muqueuse de la vésicule biliaire, qui effectue un échange sélectif d'ions inorganiques et organiques, ainsi que la fonction d'évacuation motrice, en violation de laquelle la turbulence de la voie biliaire est essentielle dans le processus de cristallisation du cholestérol et de croissance ultérieure des calculs. diminue et les conditions sont créées pour la rétention des cristaux. À la suite d'une inflammation, des microparticules pénètrent dans la lumière de la vessie, qui constituent une matrice pour le dépôt de cristaux d'une substance dans une solution sursaturée.

Mécanismes de formation des calculs biliaires pigmentaires

Les calculs pigmentaires peuvent se former sous plusieurs conditions :

Lorsque le foie est endommagé, de la bile contenant des pigments de structure anormale en est sécrétée. Ce dernier précipite, ce qui arrive avec la cirrhose du foie.

Avec la libération de pigments de structure normale, mais en quantité excessive - plus qu'il ne peut se dissoudre dans un volume de bile donné.

· Lorsque les pigments excrétés normaux sont convertis en bile en composés insolubles, ce qui peut se produire sous l'influence de processus pathologiques dans les voies biliaires.

Beaucoup reste flou sur la question de la primauté du mécanisme déclencheur de la lithiase biliaire. Récemment, malgré de nombreuses confirmations de l'hypothèse de troubles métaboliques de la formation de la bile, le rôle des facteurs locaux extrahépatiques de la lithogenèse a de nouveau été souligné. Il a été démontré qu'une diminution de la conversion enzymatique du cholestérol, une modification de la composition et du pool d'acides biliaires, la sécrétion de vésicules défectueuses en raison d'un manque d'inclusion de phospholipides dans celles-ci sont importantes, mais pas les principales raisons de la développement de la lithiase biliaire, puisque l'accélération des processus de nucléation se manifeste naturellement dans la bile kystique et non dans la bile hépatique. Le facteur le plus susceptible d'augmenter l'activité des activateurs et d'inhiber l'activité des inhibiteurs de la nucléation dans la bile de la vésicule biliaire est le processus inflammatoire dans la vésicule biliaire et l'hypersécrétion associée de glycoprotéines et de protéines, de produits de protéolyse et de peroxydation lipidique, de leucotriènes, ainsi que d'altérations métaboliques. fonction de la vésicule biliaire.

À en juger par les données expérimentales, avec toutes les méthodes d'induction de lithiase biliaire, la formation de calculs se produit dans le contexte des changements morphologiques indiqués dans la paroi de la vésicule biliaire.

TRAITEMENT

le traitement conservateur comprend :

¨ Respect du régime alimentaire au sein du 5e tableau, à savoir limiter l'utilisation d'aliments qui améliorent l'activité sécrétoire de l'estomac, du pancréas, de la sécrétion biliaire ;

¨ Prise d'antispasmodiques anticholinergiques (No-Spa, Baralgin, Spazmogard, Spazmalgin, Papaverine, Platifillin);

¨ Moyens qui régulent le péristaltisme du tractus gastro-intestinal, tels que Cerucal, Raglan;

¨ Les analgésiques sont utilisés pour réduire la douleur : antalgiques non narcotiques et antalgiques spasmo (analgine, baralgine, etc.).

Sol. Papaverini hydrochloridi- 2,0 ml X 3 fois par jour par voie intramusculaire

Sol. "No-Spa" - 2,0 ml X 3 fois par jour par voie intramusculaire

¨ Ampicilline - 1,0 ml X 4 fois par jour par voie intramusculaire

¨ Vicasoli- 1,0 ml X 3 fois par jour par voie intramusculaire

¨ Gastrocepini - 2,0 ml X 2 fois par jour par voie intramusculaire

¨ Tab. Maninili - 2 comprimés 2 fois par jour

Sol. NaCl 0,9 % - 500,0 ml

Sol. Ac. Ascorbinici 5% - 5.0 ml

Cocarboxylase - 150,0 mg

Opérationnel

Épicrise préopératoire :

La salle de bal a été admise en urgence avec des plaintes de douleurs dans l'hypochondre droit, l'érigastrie, la bouche sèche, des nausées, des vomissements et une faiblesse générale.

Examiné selon les données échographiques: Vésicule biliaire: Après avoir mangé, elle augmente à 100 * 36 mm., La paroi est épaissie jusqu'à 4 m.. Pierres dans la cavité jusqu'à 14 mm. L'un d'eux est fixé à la bouche. Le CBP n'est pas étendu.

Diagnostic clinique exposé : lithiase biliaire. Cholécystite aiguë.

En raison de la présence dans l'anamnèse d'attaques de douleur dans l'hypochondre droit pendant les repas, la présence de signes hron. cholécystite pour prévenir complications possibles et l'assainissement des voies biliaires, le patient est indiqué pour un traitement chirurgical. Cholécystectomie laparoscopique planifiée. Le patient accepte l'opération, la possibilité de conversion est avertie.

Groupe sanguin : A(II) Rh+ Lech. Médecin : N. Protocole d'opérations n° 255.

NOM ET PRÉNOM: Salmanova Alfira Fazalovna

Âge : 48 ans. Numéro de source de boulon : 22540.

Diagnostic: Lithiase biliaire. Cholécystite aiguë.

Opération: Cholécystectomie laparoscopique Drainage de la cavité abdominale.

Chirurgiens : Timerbullatov M.V., Garifullin.

Anesthésiste : A/S : Date : . Début des opérations : 9:40 . Durée: 25 minutes.

Sous anesthésie endotrachéale après traitement du champ opératoire, une laparocentèse au trocart a été introduite dans la région ombilicale. À l'aide d'un insufflateur, un carboxypéritoine tendu a été produit. Un laparoscope vidéo et des trocarts de manipulation ont été introduits dans la cavité abdominale en 4 points standards. Lors de la révision des organes abdominaux, il a été révélé que le foie n'était pas agrandi, homogène, la vésicule biliaire 8 * 4 * 4 cm n'était pas élargie, la paroi n'était pas élargie jusqu'à 4 mm avec des signes d'inflammation chronique. La vésicule biliaire a été isolée des adhérences, les éléments du triangle de Calot ont été identifiés, le canal cystique a été séparé séparément et les artères ont été suturées et sectionnées. Produit cholécystectomie du cou avec lit d'hémostase de coagulation de la vésicule biliaire. La vésicule biliaire a été évacuée par l'accès épigastrique. L'espace sous-hépatique a été aseptisé et drainé par drainage tubulaire. Contrôle de l'hémostase. Points de suture sur la plaie. Bandage aseptique. Une macropréparation - la vésicule biliaire contient des calculs de 2,5 * 10 mm., A été envoyée pour un examen histologique.

UN JOURNAL

LA DATE

L'état du patient

L'état général est relativement satisfaisant, plaintes de faiblesse, respiration vésiculeuse. La langue est sèche avec un enduit blanc, l'abdomen est mou, non enflé, indolore, la diurèse n'est pas perturbée, les selles sont formées.

Le patient après les opérations, l'état correspond aux opérations effectuées, la conscience est claire, les plaintes de douleur dans la plaie, la faiblesse, la dyspepsie, l'absence de respiration et l'hémodynamique sont satisfaisantes, l'abdomen est mou, non enflé

Le patient s'inquiète de douleurs peu intenses au niveau de l'épigastre et de l'hypochondre droit, la température est de 36,8°C. Il n'y avait pas de chaise. La miction n'est pas perturbée.

Objectivement : l'état du patient est actuellement modéré. Respiration vésiculaire dans les poumons, pouls - 78 battements par minute, rythme symétrique et régulier. BP - 130/80 mm Hg. Art. La langue est humide, recouverte d'un enduit blanchâtre. L'abdomen est modérément gonflé, ne participe pas à l'acte de respirer. La tension musculaire protectrice n'est pas détectée. La vésicule biliaire n'est pas palpable, le symptôme de Shchetkin Blumberg est négatif.

Commissaire – Zalikin M.A.

L'état du patient s'est amélioré au cours de la période écoulée. La douleur du patient n'est plus perturbée durant les 2 derniers jours. Température - 36,7 ° C. La chaise est normale. La miction n'est pas perturbée.

Objectivement : l'état du patient est actuellement satisfaisant. Respiration vésiculaire dans les poumons, pouls - 80 battements par minute, rythme symétrique et régulier. BP - 130/80 mm Hg. Art. La langue est humide, recouverte d'un enduit blanchâtre. L'abdomen est modérément gonflé, ne participe pas à l'acte de respirer. La tension musculaire protectrice n'est pas détectée. La vésicule biliaire n'est pas palpable, le symptôme de Shchetkin-Blumberg est négatif.

Commissaire – Zalikin M.A.

PRÉVOIR

Pronostic santé : récupération

Prévision pour la vie : favorable.

Prévision de travail : réalisable.

ÉPICRISE

Par étapes : poursuite du traitement à l'hôpital clinique de la ville n° 21 dans 1 département de chirurgie.

FICHE DE TEMPÉRATURE

BIBLIOGRAPHIE

1. Diagnostic des maladies les organes internes. Tome 1. A.N. Okorov. Minsk 2001.

2. Propédeutique des maladies internes. V.Kh.Vasilenko. Moscou "Médecine" 1983.

3. Classifications cliniques des maladies des organes internes. BSMU. Oufa 1996.

4. Médicaments. M.D. Mashkovsky. Moscou "Médecine" 1986.

5. Pharmacologie. D.A. Kharkevich. Moscou 2001.

6.Maladies chirurgicales Kuzin M.I. Moscou 2000

7. Rodionov V. V., Filimonov M. I., Moguchev V. M. Cholécystite calculeuse. - M. : Médecine, 1991. - 320 p.

8. Shaposhnikov A. V. Cholécystite. Pathogenèse, diagnostic et traitement chirurgical. - Maison d'édition de l'Université de Rostov, 1984. - 224 p.

9. Movchun A. A., Koloss O. E., Oppel T. A., Abdullaeva U. A. Traitement chirurgical de la cholécystite calculeuse chronique et de ses complications. - Chirurgie, 1998, n°1, p.8.

La COLIQUE RÉNALE, contrairement à la cholécystite aiguë, se caractérise par une crise aiguë de douleur dans la région lombaire avec irradiation vers l'inguine, la cuisse et des troubles dysuriques. La température reste dans la plage normale, il n'y a pas de leucocytose. Les changements abdominaux dans les coliques néphrétiques sont rares. Dans les cas graves de colique néphrétique, en particulier avec des calculs urétéraux, une distension abdominale, une tension dans les muscles de la paroi abdominale antérieure et des vomissements répétés peuvent survenir. Contrairement à la cholécystite aiguë, un symptôme Pasternatsky positif est observé et il n'y a aucun symptôme d'irritation péritonéale.

Dans l'étude de l'urine, on trouve des érythrocytes, des leucocytes, des sels.

L'APPENDICITE AIGUË avec forte localisation de l'appendice peut simuler une cholécystite.

Contrairement à appendicite aiguë la cholécystite aiguë se produit avec des vomissements répétés de bile, une irradiation caractéristique de la douleur dans la région de l'omoplate droite et de l'épaule, la région supraclaviculaire droite.

Le diagnostic est facilité par la présence dans les antécédents du patient d'indications de cholécystite ou de lithiase biliaire. L'appendicite aiguë se caractérise généralement par une évolution plus sévère avec le développement rapide d'une péritonite purulente diffuse. Symptômes d'appendicite aiguë. Souvent, le diagnostic correct est posé pendant la chirurgie.

ULCÈRE PERFORATIF DE L'ESTOMAC ET DE L'INTESTIN DE TYPE 12 (principalement des formes de perforation couvertes). Peut être diagnostiquée à tort comme une cholécystite aiguë. Par conséquent, il est nécessaire d'étudier attentivement l'anamnèse des patients. Pour la cholécystite aiguë, contrairement à un ulcère perforé, l'absence d'antécédent ulcéreux est caractéristique, la présence d'indices d'attaques antérieures de lithiase biliaire.

La cholécystite aiguë survient avec des vomissements répétés, une irradiation caractéristique de la douleur, de la fièvre et une leucocytose, ce qui n'est pas typique de la perforation de l'ulcère (une triade de symptômes).

Les perforations couvertes surviennent avec un début aigu et une tension prononcée dans les muscles de la paroi abdominale antérieure dans les premières heures après le début de la maladie; il existe souvent des douleurs locales dans la région iliaque droite dues à une fuite du contenu de l'estomac et du duodénum 12, ce qui n'est pas typique de la cholécystite aiguë. Examen radiographique, endoscopie, laparoscopie.

La PANCRÉATITE AIGUË, contrairement à l'inflammation de la vésicule biliaire, s'accompagne d'une augmentation rapide des symptômes d'intoxication, de tachycardie et de parésie intestinale. Caractérisé par une douleur dans le caractère de la ceinture épigastrique, accompagnée de vomissements fréquents, parfois indomptables.

Le diagnostic est facilité par la présence d'une teneur accrue en diastase dans les urines et le sang et d'une hyperglycémie, caractéristique de la pancréatite aiguë. Symptômes de pancréatite.

Le diagnostic différentiel présente de grandes difficultés (théorie du « canal unique »).

LA DYSKINESIE DES VOIES BILIAIRES procède avec température normale, l'état des patients est satisfaisant, il n'y a pas de tension dans les muscles de la paroi abdominale antérieure et des symptômes d'irritation péritonéale. Analyse du sang et de l'urine sans changement.

La colique biliaire, contrairement à la cholécystite aiguë, se caractérise par une crise aiguë de douleur, sans fièvre ni hyperleucocytose. Après une crise, les patients ne présentent généralement pas de tension dans les muscles de la paroi abdominale antérieure et des symptômes d'irritation péritonéale, typiques de la cholécystite aiguë. Il convient de rappeler qu'après une attaque de colique biliaire, une cholécystite aiguë sévère peut se développer et, par conséquent, un traitement chirurgical sera nécessaire.

Dans ces cas, après une attaque de colique biliaire, la douleur dans l'hypochondre droit persiste et l'état des patients s'aggrave. Il y a une augmentation de la température, une leucocytose, une tension dans les muscles de la paroi abdominale antérieure et une douleur à la palpation dans l'hypochondre droit.

PNEUMONIE DU CÔTÉ DROIT.

INFARCTUS DU MYOCARDE. La pathologie cardiaque est de nature réflexe et disparaît après la guérison de la cholécystite. La douleur dans la région du cœur avec cholécystite est appelée syndrome cholécystocardique de Botkin.

Le diagnostic différentiel entre l'infarctus du myocarde et la cholécystite n'est pas une tâche facile lorsque, en plus des symptômes de la cholécystite aiguë, il existe des symptômes de lésions du muscle cardiaque et que les données ECG n'excluent pas une crise cardiaque. L'échographie et la laparoscopie diagnostique sont d'une grande importance, ce qui nécessite un soutien anesthésique spécial et un pneumopéritoine strictement contrôlé afin de ne pas compliquer encore plus le travail du cœur.

Si un patient a une cholécystite compliquée d'ictère, il faut diagnostic différentiel la jaunisse, qui se caractérise par une augmentation du taux de bilirubine dans le sang. Il existe trois principaux types de jaunisse.

L'ictère hémolytique (préhépatique) survient à la suite d'une dégradation intensive des globules rouges et d'une production excessive de bilirubine indirecte. Raison - l'anémie hémolytique associée à un hyperfonctionnement de la rate dans l'hypersplénisme primaire et secondaire. Dans ce cas, le foie n'est pas capable de faire passer une grande quantité de bilirubine à travers la cellule hépatique (bilirubine indirecte). La peau est jaune citron, il n'y a pas de prurit. Il y a pâleur en combinaison avec la jaunisse. Le foie n'est pas hypertrophié. L'urine est de couleur foncée, les matières fécales sont intensément colorées. Il y a une anémie, une réticulocytose.

Jaunisse parenchymateuse (hépatique) - hépatite virale, cirrhose du foie, intoxication par certains poisons hépatotropes (tétrachloroéthane, arsenic, composés phosphorés). Des dommages aux hépatocytes se produisent, la capacité des cellules hépatiques à lier la bilirubine libre et à la convertir en diminution directe. La bilirubine directe ne pénètre que partiellement dans les capillaires biliaires, une partie importante de celle-ci retourne dans le sang.

La maladie a une période prodromique prononcée sous forme de faiblesse, manque d'appétit, légère fièvre. Il y a des douleurs sourdes dans l'hypochondre droit. Le foie est hypertrophié et compacté. La peau est jaune safran avec une teinte rubis. Dans le sang, le niveau de bilirubine directe et indirecte, les aminotransférases est augmenté, la concentration de prothrombine diminue. Les matières fécales sont colorées. Mais dans les hépatites virales sévères au plus fort de la maladie, avec des dommages importants aux cellules du foie, la bile peut ne pas pénétrer dans l'intestin, les matières fécales seront alors acholiques. Avec la jaunisse parenchymateuse, les démangeaisons sont faibles.

Pour clarifier le diagnostic de l'échographie, la laparoscopie.

L'ictère obstructif (sous-hépatique, obstructif) se développe en raison de l'obstruction des voies biliaires et de l'altération du passage de la bile dans l'intestin. La raison en est les calculs dans le canal, le cancer de la vésicule biliaire avec passage au cholédoque, le cancer de la membrane muqueuse du canal lui-même, l'OBD, la tête pancréatique, les métastases tumorales d'une autre localisation dans la porte du foie ou la compression des canaux par une tumeur de l'estomac.

Les causes rares sont les rétrécissements cicatriciels des conduits, les vers ronds dans la lumière des conduits, la lymphadénite péricholédoque, la ligature des conduits lors d'une intervention chirurgicale.

La peau est vert-jaune, parfois jaune-gris. Prurit persistant. L'obstruction des canaux entraîne une hypertension biliaire, qui affecte négativement le parenchyme hépatique. Avec l'ajout de cholangite, de la fièvre est observée. L'urine du patient est de couleur foncée, les matières fécales sont acholiques. Dans le sang - une augmentation de la teneur en bilirubine directe. ultrason. ChPH.

Complications de la cholécystite

CHOLEDOCOLITHOSE.

STÉNOSE DE BDS.

La cholangite est une inflammation aiguë ou chronique des voies biliaires. C'est une complication redoutable qui entraîne une intoxication sévère, un ictère, une septicémie. Désintoxication. Antibiothérapie.

Fistule cholécystoduodénale - l'attaque est autorisée, cependant, le reflux du contenu de l'intestin dans la vésicule biliaire contribue à la poursuite des phénomènes d'inflammation de la paroi de la vessie. Pierres dans l'intestin - obstruction intestinale obstructive.

11. Traitement de la cholécystite (schéma)

CONSERVATEUR. Hospitalisation au service de chirurgie. Repos au lit. Exclusion de la nutrition entérale (eau minérale). Froid au ventre. Lavage gastrique à l'eau froide. thérapie par perfusion. Antispasmodiques. Analgésiques. Antihistaminiques. Si la douleur n'est pas soulagée - promedol. Omnopon et la morphine ne doivent pas être prescrits - ils provoquent des spasmes du sphincter d'Oddi et de Lutkens. Blocage de la novocaïne du ligament rond du foie.

cholécystite obstructive.

La séquence de développement des changements locaux comprend les composants suivants:

1) obstruction du canal cystique ;

2) une forte augmentation de la pression dans la vésicule biliaire;

3) stase dans les vaisseaux de la vésicule biliaire ;

4) bactériocholie ;

5) destruction de la paroi vésicale ;

6) s'infiltrer ;

7) péritonite locale et diffuse.

Cholécystite aiguë

Conserves simples compliquées. traitement,

(hypertension biliaire) examen (simple)

obturation cholécystite Avec hypertension Chirurgie élective

conduits (CE, LCE, MCE)

Déblocage Hydropisie Sténose Destructrice OBD Cholédo-

vésicule biliaire g. cholécystite vésicale lithiase

Groupe de mise à niveau des opérations planifiées Opération urgente Jaunisse Cholan-

(ChE) risque talkie-walkie (ChE,LHE,MHE) git

Préopératoire Sortie de l'Opération en urgence

préparation de la vessie d'affilée (ChE, choledocholi-

totomie, PSP, drainage en T,

RPHG, EPST, LHE, MHE

Le processus peut se développer dans trois directions :

1. Libération de bulles. Dans ce cas, le traitement est poursuivi jusqu'à la disparition complète des phénomènes aigus, puis le patient est examiné pour identifier les calculs, l'état de la vésicule biliaire, etc.

2. Hydropisie de la vésicule biliaire - avec une infection peu virulente ou son absence, avec la capacité préservée de la paroi de la vessie à s'étirer davantage. La douleur et la réaction périfocale disparaissent. Pendant longtemps, une telle bulle peut ne pas déranger, mais tôt ou tard une exacerbation se produit. En raison de ce danger, l'hydropisie de la vessie est une indication directe d'une opération planifiée.

3. Cholécystite destructrice. Si le traitement conservateur échoue, le déblocage ne s'est pas produit et un processus infectieux se développe dans la vésicule biliaire, qui se manifeste par une augmentation de la température corporelle, une leucocytose et l'apparition de symptômes d'irritation péritonéale, cela signifie alors l'apparition d'une cholécystite destructrice. (phlegmoneux ou gangreneux). Le processus dans ce cas devient incontrôlable et dicte l'adoption des mesures les plus urgentes.

Si dans les 24 à 48 heures avec un traitement conservateur continu, il n'y a pas de déblocage de la vessie, il est alors nécessaire de vérifier la présence d'une cholécystite destructrice chez le patient.

Traitement de la cholécystite obstructive (conservateur et chirurgical).

CHIRURGICAL.

Par heure:

Une opération d'urgence est effectuée immédiatement après l'entrée du patient à l'hôpital ou après une préparation vitale à court terme, qui ne prend pas plus de quelques heures. L'indication est la péritonite.

Chirurgie précoce (24-72 heures) - avec l'inefficacité du traitement conservateur, ainsi qu'en cas de cholangite, ictère obstructif sans tendance à les éliminer, en particulier chez les patients âgés et séniles ;

Tardif (prévu) - 10-15 jours et plus tard après la disparition de la cholécystite aiguë.

1. Préparation préopératoire.

2. Soulagement de la douleur.

3. Accès. Coupes de Kocher, Fedorov, Kera, Rio Branco, laparotomie médiane.

Toutes les maladies des voies biliaires sont réparties dans les groupes suivants:

1) dyskinésie biliaire ;

2) maladies inflammatoires - cholécystite, cholangite;

3) lithiase biliaire ;

4) syndrome biliaire-cardiaque ;

5) syndrome post-cholécystectomie ;

6) tumeurs des voies biliaires - bénignes et malignes.

Les patients atteints de maladies des voies biliaires sont très fréquents. Leur nombre est en augmentation et au moins le double du nombre de patients souffrant d'ulcère peptique.

9.1. Dyskinsie des voies biliaires- troubles fonctionnels du tonus et de la motricité de la vésicule biliaire et des voies biliaires. Il existe un hypertonique-hyperkinétique, caractérisé par un état hypertonique de la vésicule biliaire (généralement en combinaison avec une hypertonicité des sphincters de Lutkens et d'Oddi), et un hypotonique-hypokinétique, caractérisé par un état hypotonique de la vésicule biliaire et du sphincter d'Oddi. Ils sont plus fréquents chez les femmes. La dyskinésie hypertonique-hyperkinétique est plus souvent détectée à un âge plus jeune, et hypotonique-hypokinétique - chez les asthéniques et chez les personnes de plus de 40 ans. La dyskinésie des voies biliaires est principalement due à une violation de la régulation neurohumorale, survient avec la névrose, le syndrome diencéphalique, le solarium, les troubles endocriniens-hormonaux (avec hypo- et hyperthyroïdie, ménopause, fonction insuffisante des ovaires, glandes surrénales et autres glandes endocrines) . La forme hypertensive-hyperkinétique survient par réflexe (avec ulcère peptique, colite, appendicite, annexite, etc.), en violation de la production de cholécystokinine-pancréozymine par la muqueuse duodénale (duodénite atrophique), après une maladies infectieuses, hépatite virale, due au béribéri, dénutrition, intoxications endogènes et exogènes, repas fréquents mais irréguliers, passion excessive pour les aliments épicés qui irritent la muqueuse du tube digestif. La forme hypotonique-hypokinétique de la dyskinésie est détectée avec une constitution asthénique prononcée, un mode de vie sédentaire, une mauvaise alimentation avec de très longs intervalles entre les repas. En plus de ce qui précède, la dyskinésie de la vésicule biliaire et des voies biliaires s'accompagne de lésions organiques du système biliaire (cholécystite, cholangite, lithiase biliaire, etc.), d'invasions parasitaires et helminthiques du tube digestif.

Clinique.

Dyskinésie hypertensive-hyperkinétique voies biliaires se produit avec des douleurs paroxystiques dans l'hypochondre droit d'un caractère douloureux ou crampes, rappelant les crises de coliques dans la lithiase biliaire. Des troubles, un stress neuropsychique important, des émotions négatives contribuent à la survenue de crises ; chez la femme, des exacerbations peuvent être associées au cycle menstruel. La douleur peut irradier vers l'omoplate droite, l'épaule droite, parfois vers la région du cœur, accompagnée de réactions autonomes générales - transpiration intense, pâleur, nausées, parfois maux de tête, palpitations, etc. Les crises de douleur durent de quelques minutes à plusieurs jours ; pressage sourd et C'est une douleur sourde dans l'hypochondre droit peut persister pendant des semaines, s'intensifiant ou s'atténuant parfois. Lors d'une crise douloureuse, l'abdomen n'est pas tendu. La jaunisse, la réaction fébrile, la leucocytose et l'augmentation de la VS après une attaque ne sont pas observées.

Forme de dyskinésie hypotonique-hypokinétique se manifestant principalement par des douleurs de faible intensité dans l'hypochondre droit, souvent prolongées. La palpation a révélé une légère douleur dans la vésicule biliaire. Diagnostic confirmé par un sondage duodénal chromodiagnostique en plusieurs étapes ; avec la forme hypertonique-hyperkinétique de la dyskinésie, la phase II (sphincter fermé d'Oddi) peut être allongée, la phase IV (contraction de la vésicule biliaire) peut être raccourcie. Dans la forme hypotonique-hypokinétique de dyskinésie, la phase II peut être absente (le sphincter d'Oddi est constamment dans un état détendu), la phase IV est retardée et allongée; souvent un réflexe kystique ne peut être obtenu qu'avec l'introduction d'un stimulus suffisamment fort (cholécystokinine-pancréozymine). Cholecysto- et cholégraphie intraveineuse orale, échographie, dans les cas plus difficiles au diagnostic - tomodensitométrie confirmer le caractère fonctionnel de la maladie. Avec la forme hypertonique-hyperkinétique, une contraction accélérée et forte de la vésicule biliaire (de plus des 2/3 du volume d'origine) est déterminée par radiographie en série après l'introduction d'un stimulant (deux jaunes d'œufs), avec la forme hypotonique-hypokinétique de la vésicule biliaire grandes tailles, dans certains cas - omis, réduit lentement (moins de 1/3 du volume d'origine). L'état hypotonique du sphincter d'Oddi est parfois détecté lors d'un examen radiographique du duodénum (écoulement de la masse de contraste à travers le sphincter détendu d'Oddi dans la voie biliaire principale). Récemment, la cholécystographie radio-isotopique a été utilisée. Couler généralement longue avec des périodes d'exacerbations (souvent provoquées par un stress émotionnel, des troubles nutritionnels) et des rémissions.

Traitement. Régulation de l'alimentation, bonne alternance de travail et de repos, normalisation du sommeil et d'autres fonctions du système nerveux central (sédatifs, tranquillisants, procédures de physiothérapie).

À forme hypertonique-hyperkinétique en période d'exacerbation, les régimes n ° 5, 5a et 2 sont indiqués - magnésium (avec limitation des irritants alimentaires mécaniques et chimiques, graisses), médicaments antispasmodiques et anticholinergiques, procédures de physiothérapie thermique, eau minérale à faible minéralisation (Slavyanskaya et Smirnovskaya, Essentuki n ° 4 et n ° 20, narzan et autres sont généralement chauds, fractionnés, 5 à 6 doses par jour).

À dyskinésie hypotonique-hypokinétique régime recommandé numéro 5, 15 ou 3, stimulants psychomoteurs et analeptiques (préparations de strychnine, caféine, etc.), physiothérapie et agents physiothérapeutiques à action tonique, sondages duodénaux répétés, tubes fermés, traitement de cours avec de l'eau minérale à haute minéralisation telle que Essentuki n ° 17, Arzni, Batalinskaya, etc.; l'eau est prescrite sous une forme froide ou légèrement chaude, 200-250 ml 2-3 fois par jour 30-90 minutes avant les repas.

Dans les dyskinésies réflexes des voies biliaires, le traitement de la maladie sous-jacente est d'une importance primordiale.

9.2. Cholécystite- Maladie inflammatoire de la vésicule biliaire. La cholécystite est aiguë et chronique (la durée de la maladie est supérieure à 6 mois), alithiasique et calculeuse.

9.2.1. Cholécystite aiguë(cm. Syndrome abdominal aigu).

9.2.2. Cholécystite chronique peut être le résultat d'une cholécystite aiguë non traitée, mais se développe le plus souvent progressivement, comme une maladie chronique primaire. La cholécystite alithiasique chronique est souvent stade initial cholécystite calculeuse. Le rôle principal dans le développement de la cholécystite chronique est joué par une infection qui pénètre dans la vésicule biliaire à partir d'autres foyers d'infection (amygdalite chronique, maladie parodontale, pyélite, etc.) par voie hématogène (par l'artère hépatique ou la veine porte) ou lymphogène. L'infection la plus courante de la bile se produit en remontant de l'intestin. Dans la plupart des cas, les agents responsables sont Escherichia coli, streptocoques, staphylocoques, entérocoques, proteus. Le développement et la progression du processus inflammatoire sont facilités par une diminution de la réactivité du système hépatobiliaire, une alimentation déséquilibrée, une stagnation de la bile dans la vésicule biliaire, une sensibilisation de l'organisme, un surpoids, une constipation, etc.

Manifestations cliniques la cholécystite chronique est associée à un certain nombre de facteurs : l'intensité et la durée du processus inflammatoire dans la vésicule biliaire et les voies biliaires extrahépatiques, concomitantes troubles fonctionnels système biliaire, implication dans le processus pathologique d'autres organes digestifs. Premier symptôme clinique- douleur dans l'hypochondre droit et moins souvent dans l'épigastre, souvent de nature douloureuse, n'atteignant pas une grande intensité. Son équivalent peut être une lourdeur ou une brûlure de la même localisation. En présence de dyskinésie hypertensive, la douleur est intense, de courte durée, de nature paroxystique. Les raisons de son apparition peuvent être non seulement des erreurs dans le régime alimentaire (réception d'aliments gras et frits, des collations froides, des œufs, des boissons gazeuses), mais également une surcharge émotionnelle. Si dans le premier cas la douleur est associée à un étirement de la vésicule biliaire, alors dans le second - à un spasme musculaire. La douleur peut également être due à une péricholécystite. Dans ce cas, il est constant, répandu (déterminé dans toute la zone de l'hypochondre droit), augmente avec les virages et les inclinaisons du corps. Il peut y avoir des vomissements mélangés à de la bile, de l'amertume dans la bouche et des éructations "amères". En violation de la sécrétion biliaire, des démangeaisons cutanées sont possibles. Un ictère transitoire peut parfois survenir avec un spasme prolongé du sphincter d'Oddi ou en raison de la migration de la pierre dans le canal cholédoque et de l'obstruction de l'écoulement de la bile. Il y a souvent une irritabilité accrue, de la fatigue, de l'asthénie, une diminution de l'appétit, des brûlures d'estomac, des éructations, des flatulences, de la constipation, de la diarrhée, etc. La palpation révèle une douleur dans l'hypochondre droit (projection de la vésicule biliaire), des symptômes positifs de Grekov-Ortner, Kera, Georgievsky- Mussi (phrenicus -symptôme), etc., les zones de sensibilité cutanée accrue (symptômes de Jonas, Boas) sont déterminées. Dans le test sanguin général - leucocytose avec un changement de coup, une augmentation de la RSE (plus significativement avec une cholécystite calculeuse). Cependant, la cholécystite chronique peut évoluer pendant des années comme une maladie lente avec une image normale du sang périphérique. Sondage duodénal et examen de la bile: turbidité, grand nombre de flocons, épithélium cylindrique desquamé, mucus avec présence de leucocytes imbibés de bile, déplacement du pH vers le côté acide, augmentation des propriétés lithogènes, etc. sont détectés dans la partie kystique (B). importance attaché à l'étude échographique (tomodensitométrie) et radiologique (cholécystocholangiographie) du système des voies biliaires. À l'aide de ces méthodes, il est possible d'identifier la déformation, l'augmentation ou la diminution de la vésicule biliaire, l'épaisseur de la paroi, les adhérences, l'expansion et le rétrécissement des conduits, de diagnostiquer les calculs dans la vessie et les conduits, etc. La culture biliaire aide à déterminer la microflore et sa sensibilité aux antibiotiques.

Traitement et prévention. Avec l'exacerbation de la cholécystite, les mesures thérapeutiques visent à éliminer le processus inflammatoire, la stagnation de la bile, les troubles métaboliques et la dyskinésie biliaire. Les patients se voient prescrire un repos au lit ou un semi-lit (jusqu'à ce que la douleur disparaisse), un régime d'épargne (n ° 5), en l'étendant progressivement à mesure qu'ils se sentent mieux, des repas fractionnés.

Avec l'exacerbation de la cholécystite bactérienne, les antibiotiques sont indiqués (la sensibilité de la microflore biliaire aux antibiotiques doit être prise en compte), principalement un large éventail actions (ampiox 1 g 4 fois par jour par voie intramusculaire ou 0,5 g par voie orale 4 à 6 fois par jour, dicloxacilline 0,5 à 1 g 4 fois par jour par voie orale une heure avant les repas ou 1,5 heure après la prise de nourriture, céfobide 1 à 2 g 2 fois par jour, tarivid 1 comprimé - 0,2 g 2 fois par jour, klaforan 1 g 2 fois par jour, unazine, composé de sulbactam et d'ampicilline, 1,5 g chacun 3 fois par jour par voie intramusculaire, etc.), abaktal, 0,8 g par jour, bactrim ou biseptol, 2 comprimés 2 fois par jour après les repas, etc. Pour la cholécystite à Giardia, la furazolidone est prescrite (0,1 g 3 à 4 fois par jour), le métronidazole (0,25 g 3 fois par jour), la fazizhin (2 g par jour) . La durée du traitement est de 5 à 7 jours, suivie d'une étude de la bile. La thérapie antibactérienne (cours 8-12 jours) est associée à des vitamines et à un cholérétique. Ces derniers sont utilisés en fonction du type de dyskinésie biliaire concomitante. Avec la dyskinésie hypomotrice, la cholécystocinétique (sulfate de magnésium, sel de Karlovy Vary, xylitol, sorbitol, huile d'olive, pituitrine, oliméthine, etc.) est indiquée, et avec la dyskinésie hypertensive - cholérétiques (allochol, chologon, cholenzyme, oxafénamide, tsikvalon, tanacechol, flamin , holosas et etc.). Il est impossible d'utiliser des cholérétiques avec un processus inflammatoire prononcé dans le système des voies biliaires, une hépatite et une cholékinétique - avec hypertension duodénale et biliaire, hollithiase. Pour éliminer le syndrome douloureux, des antispasmodiques sont administrés par voie parentérale: papavérine (2 ml d'une solution à 2%), no-shpu (2 ml d'une solution à 2%), halidor (2 ml d'une solution à 2,5%), métoclopramide (raglan, cerucal) 2 ml chacun, atropine (1 ml d'une solution à 0,1 %), méthacine (2 ml d'une solution à 0,1 %), platyfiline (2 ml d'une solution à 0,2 %). En cas de syndrome douloureux sévère, analgin (2 ml d'une solution à 50%) ou fortral (1-2 ml d'une solution à 3%), promedol (1 ml d'une solution à 2%) ou baralgin (5 ml par voie intraveineuse ou intramusculaire) , thalamonal (2-4 ml dans le muscle). Un bon effet antispasmodique sur les sphincters de Lutkins et l'ampoule hépatique-pancréatique (sphincter d'Oddi) est fourni par les médicaments de la série des nitroglycérines (nitroglycérine 1 capsule ou comprimé sous la langue). Les médicaments énumérés sont prescrits 3 à 4 fois par jour et, à mesure que la douleur diminue, ils passent à la prise de médicaments avec action similaireà l'intérieur. La présence de bile lithogénique est une indication de utilisation à long terme herbes médicinales à effet cholérétique (immortelle, millepertuis, stigmates de maïs, menthe poivrée, tanaisie, nikodine, etc.), hydrocholérétiques (eaux minérales, sel de Karlovy Vary, acide chlorhydrique, salicylates). Avec une vésicule biliaire congestive, il est nécessaire de faire un sondage thérapeutique duodénal ou un tubage 1 à 2 fois par semaine. Pour augmenter le tonus de la vésicule biliaire, une infusion de racine de ginseng, radiola rosea (racine dorée), éleuthérocoque, fruit de Schisandra chinensis est prescrite. Après l'arrêt actif processus inflammatoire la physiothérapie est utilisée avec succès: procédures thermiques sur la région du foie (chauffage, applications de paraffine, ozocérite, tourbe), champ électrique UHF, thérapie par micro-ondes, courants modulés sinusoïdaux, magnétothérapie, hydrothérapie (bains de conifères, de perles, d'oxygène et autres), thérapie par l'exercice , acupuncteur. Le traitement chirurgical est indiqué chez les patients atteints de cholécystite calculeuse, il ne faut pas s'attendre à des complications (ictère obstructif, cholangite, empyème de la vésicule biliaire, pancréatite, hépatite, etc.). Le développement de complications est lecture absolue pour retirer la vésicule biliaire et les calculs. À réhabilitation les patients jouent un rôle important dans le traitement des sanatoriums (sanatoriums "Druskininkai", "Krinitsa", "Porechye", "Naroch", "Pridneprovsky", etc.).

9.3. Cholangite- inflammation des voies biliaires. Ça arrive aiguë et chronique, primaire et secondaire. La cholangite secondaire est beaucoup plus fréquente. Cause cholangite - une infection bactérienne : E. coli, streptocoques, staphylocoques, entérocoques, bactéries anaérobies, protozoaires qui pénètrent dans les voies biliaires par la circulation sanguine ou remontent de l'intestin en raison du reflux du contenu du duodénum. Le développement du processus inflammatoire est facilité par la stagnation de la bile (blocage du canal cholédoque par une pierre, rétrécissement du canal cholédoque, compression du canal de l'extérieur, tête du pancréas ou ganglions lymphatiques hypertrophiés, etc.). Le plus souvent, la cholangite complique l'évolution de la cholécystite calculeuse. Par la nature du processus, on distingue les cholangites catarrhales, purulentes et nécrotiques.

Clinique la cholangite dépend de l'agent causal de la maladie et de la nature du processus pathologique.

Cholangite aiguë se manifeste par de la fièvre, des frissons, des sueurs abondantes, des douleurs dans l'hypochondre droit, des troubles dyspeptiques, une faiblesse. À l'avenir, l'hépatite se développe, le foie grossit, douloureux à la palpation. Bientôt rejoint la splénomégalie. Typiquement, l'apparition d'un ictère, une leucocytose avec un déplacement prononcé de la formule leucocytaire vers la gauche et une augmentation de la VS. À partir d'une portion de bile C, dans la plupart des cas, l'agent causal de la maladie est semé. Parmi les cholangites aiguës, il y a forme de rechute(se produit avec un calcul valvulaire dans la section terminale du canal cholédoque) avec des frissons périodiques et de la fièvre. Chez les personnes âgées, une cholangite purulente se développe souvent, qui peut être compliquée par une péritonite, un abcès sous-diaphragmatique, un empyème pleural et une insuffisance hépatique-rénale.

Cholangite chronique peuvent se développer principalement ou être la conséquence d'une cholangite aiguë. Attribuer la forme latente, récurrente et septique, ainsi que la cholangite sclérosante primitive. Dans la forme latente, la douleur et la sensibilité lors de la palpation dans l'hypochondre droit sont absentes ou légères. Une faiblesse, une température parfois subfébrile, des frissons, parfois des démangeaisons, une légère leucocytose et une VS élevée sont observées. Le foie augmente progressivement et, dans ce contexte, une cirrhose biliaire secondaire du foie peut se former.

Pour forme récurrente caractérisé par des crises de douleur récurrentes avec une augmentation de la température, des frissons, de la transpiration et d'autres manifestations de cholangite.

forme septique l'angiocholite chronique est rare. Provoque son streptocoque vert. La maladie est grave, accompagnée d'une bactériémie, d'une hypertrophie de la rate, de lésions rénales.

Cholangite sclérosante primitive (sténosante). Les raisons de son développement ne sont pas entièrement comprises. Dans certains cas, les voies biliaires principalement extrahépatiques sont plus souvent touchées, dans d'autres - intrahépatiques. Les principaux symptômes sont la jaunisse et le prurit. Aux stades ultérieurs, des signes de cirrhose biliaire du foie sont révélés.

Diagnostic il est vérifié par une modification de l'image sanguine (leucocytose modérée, augmentation de la VS, granularité parfois toxique des neutrophiles), la détection d'éléments inflammatoires et de microbes dans la partie C de la bile, les données des rayons X, des ultrasons et d'autres études de le système hépatobiliaire. La cholangiographie et la cholédochoscopie ont une importance déterminante dans le diagnostic.

Traitement. Antibiothérapie active (ampiox, dicloxacilline, ofloxacine, klaforan, etc., cf. Traitement de la cholécystite) en tenant compte de la sensibilité de la microflore à ceux-ci, de l'oxygénation hyperbare, de la désintoxication, des vitamines, etc., et en cas d'obstruction des voies biliaires - traitement chirurgical. Dans la cholangite chronique, en plus de l'antibiothérapie, des antispasmodiques, cholérétiques, tubazh, des préparations multivitaminées sont utilisés, et dans la cholestase avec démangeaisons, des sorbants d'acide biliaire (cholestyramine, bilignine, entérodèse, etc.).

9.4. Maladie des calculs biliaires (GSD). Il s'agit d'une maladie du système hépatobiliaire avec formation de calculs biliaires dans la vésicule biliaire, et parfois dans les voies biliaires. La maladie se développe à la suite d'un certain nombre de facteurs: stagnation de la bile à la suite d'un repas rare, troubles métaboliques, modifications de la structure de la bile (dyscholie), processus infectieux-inflammatoires et dystrophiques du système hépatobiliaire, inactivité physique, consommation excessive de graisses animales et de viande, troubles circulatoires dans la paroi de la vésicule biliaire (athérosclérose des artères de la vessie), prédisposition héréditaire, diabète sucré. Par composition chimique, les calculs biliaires sont divisés en cholestérol, pigment, calcaire (calcium) et mixte. calculs de cholestérol se composent principalement (60 % ou plus) de cholestérol avec un mélange de protéines, de chaux et de pigments. Ils ont souvent une forme arrondie, lisse, légère, de couleur jaune blanchâtre, flottent dans l'eau. Mécanisme de formation des calculs de cholestérol : Le cholestérol est sécrété dans la bile presque exclusivement dans la composition des micelles formées par les acides biliaires et les vésicules phospholipidiques, ce qui lui permet d'être à l'état dissous. Une quantité excessive de cholestérol et une teneur réduite en phospholipides et en acides biliaires créent les conditions préalables pour que le cholestérol tombe de la solution et forme des calculs. pierres pigmentaires se forment en raison de la stagnation de la bile dans le foie, les voies biliaires extrahépatiques, ainsi que dans l'hémolyse, la cirrhose du foie et l'hyperbilirubinémie enzymatique. Ils comprennent des pigments biliaires, des protéines et une petite quantité de sels de chaux. Ils sont généralement multiples, petits, brun-noir, mous. pierres de chaux sont rares. Ils se forment sur fond d'inflammation, denses, de couleur blanchâtre, bosselés. Les plus courants sont les calculs mixtes cholestérol-pigment-calcaire. Ils sont multiples, peuvent varier en taille du grain de mil au œuf de poule et se forment principalement dans le contexte du processus inflammatoire de la vésicule biliaire et des conduits.

Tableau clinique et diagnostic. On distingue les variantes suivantes de l'évolution de la lithiase biliaire: 1) une forme latente de lithiase biliaire (porteur de calculs), lorsque les calculs ne se manifestent pas cliniquement et sont diagnostiqués par hasard; 2) lithiase biliaire - la migration des calculs dans la vésicule biliaire et les conduits provoque des douleurs et d'autres troubles ; 3) cholécystite calculeuse - la forme la plus grave de lithiase biliaire, lorsque le syndrome douloureux (généralement sous forme de colique biliaire) est associé à des signes d'inflammation active de la vésicule biliaire et des conduits.

À forme latente Le patient GSD ne se plaint pas de la vésicule biliaire. La sécrétion biliaire n'est pas perturbée, l'étude du système hépatobiliaire est indolore. La formule sanguine complète et les tests de la fonction hépatique n'ont pas été modifiés. Le diagnostic est établi à l'aide de méthodes de recherche spéciales (échographie, méthodes radio-opaques).

GSD se produit avec une grande variété de manifestations cliniques. La douleur la plus typique est de nature paroxystique (coliques biliaires) dans l'hypochondre droit et l'épigastre avec irradiation de l'omoplate droite, de l'épaule droite et, plus précisément, de la moitié droite de la poitrine, en particulier avec le développement ultérieur d'une hyperbilirubinémie, d'une décoloration des matières fécales et assombrissement de l'urine. Les crises de douleur peuvent être très fréquentes (quotidiennes) ou 1 à 2 fois par an. la douleur aiguë dans l'hypochondre ou l'épigastre droit dure des heures, et parfois plus d'une journée, les vomissements inquiètent, ces phénomènes ne sont éliminés qu'après l'introduction de médicaments antispasmodiques et de médicaments. Une crise de douleur est provoquée par la consommation d'aliments gras, d'alcool, de mouvements brusques, d'activité physique, d'émotions négatives, etc. Souvent, la colique biliaire survient sans raison apparente. Lors d'une crise, l'abdomen est modérément tendu, douloureux à la palpation dans l'hypochondre droit (dans la zone de la vésicule biliaire), symptômes positifs de Kerr, Ortner, Murphy, etc. En dehors de la crise, ces symptômes sont légers voire absents. Chez les patients atteints de lithiase biliaire, les crises typiques de colique biliaire ne sont souvent pas observées et la douleur est sourde, constante ou intermittente. Les patients sont plus préoccupés par les nausées, l'amertume dans la bouche, la diarrhée ou le relâchement des selles après avoir mangé des aliments gras (c'est la forme dite dyspeptique). Les calculs de la vésicule biliaire peuvent migrer dans le canal cystique et le bloquer. Avec un blocage complet du canal cystique ou du col de la vésicule biliaire, les restes de bile de la vessie déconnectée sont absorbés et sa cavité est remplie d'un liquide jaunâtre - une hydropisie de la vésicule biliaire se produit ou la vésicule biliaire rétrécit et sa cavité est oblitérée. Les complications de la lithiase biliaire comprennent également la cholangite, la jaunisse obstructive, la pancréatite, l'hépatite, les escarres et la perforation de la vésicule biliaire, les fistules internes, la péricholécystite. Développe souvent un syndrome biliaire-cardiaque. Diagnostique. Le sondage duodénal révèle une modification de la structure de la bile kystique (partie B), et dans certains cas hépatique (partie C) : une diminution de la teneur en phospholipides, en particulier les acides biliaires, et une augmentation de la quantité de cholestérol (une diminution du coefficient lithogène de plus de moitié), la détermination d'un grand nombre de cristaux de cholestérol et de bilirubinate de calcium (sable biliaire). Pour visualiser les calculs, des méthodes de recherche radio-opaques et échographiques, la cholangiocholécystopancréatographie ascendante endoscopique sont utilisées. Les méthodes de contraste aux rayons X permettent de diagnostiquer une vésicule biliaire invalide.

Cholécystite calculeuse- la variante la plus sévère de l'évolution de cette maladie (voir. Cholécystite chronique).

Traitement. La prévention de la lithiase biliaire doit être réalisée en présence de facteurs de risque de lithiase biliaire: surpoids, notamment dû à une alimentation hypercalorique; prédisposition héréditaire aux maladies associées aux troubles métaboliques (obésité, lithiase urinaire, goutte, etc.); les maladies du système digestif, ainsi que l'athérosclérose, Diabète; grossesses fréquentes, troubles métaboliques pendant la grossesse et stase biliaire; surcharge neuropsychique et émotionnelle; les troubles endocriniens, notamment chez la femme (ménopause, hypothyroïdie, etc.) ; hypokinésie; hépatite virale transférée, etc.

Dans ces cas, il est important d'avoir une alimentation équilibrée avec l'inclusion d'une quantité suffisante de fibres alimentaires dans l'alimentation, en observant le rythme de l'alimentation, en excluant les aliments frits et en limitant les aliments contenant du cholestérol, en effectuant des cures de thérapie diététique de décharge et en prescrivant un régime de déchargement une fois par semaine (légumes, fruits, pastèque, kéfir - sélectionnés individuellement).

En présence de petits calculs flottants dans la vésicule biliaire, dont la taille ne dépasse pas le diamètre du canal vésiculaire, des agents cholérétiques à action cholékinétique sont indiqués (oliméthine, sulfate de magnésium, xylitol, sorbitol, mannitol, les huiles végétales etc.) ainsi que des antispasmodiques (no-shpa, papaverine, halidor.metoclopramide, platifilline, eufillin, etc.), des tubages, des médicaments qui améliorent les processus métaboliques et la microcirculation, y compris les hépatoprotecteurs (riboxine 0,2 g 3 fois par jour ou goutte à goutte intraveineuse, cocarboxylase , 0,1 g goutte à goutte intraveineux dans une solution saline ou de glucose à 5 %, heptral, 0,4 g 3 fois à l'intérieur avant les repas ou 0,8 g goutte à goutte intraveineux, legalon 2 comprimés 3 fois par jour avant les repas, acide lipoïque 0,025 g 3 fois par jour à l'intérieur, hepatofalk 1 comprimé 3 fois par jour avant les repas…), phytocholérétiques (flamin, fleurs d'immortelle des sables, bourgeons de bouleau, fleurs de bleuet bleu, tanacechol…).

Pour soulager la douleur (arrêt des coliques biliaires), en plus des antispasmodiques myotropes, analgin (2 ml d'une solution à 50%) est administré par voie intramusculaire ou intraveineuse ou le mélange dit lytique (analgin, papaverine, diphenhydramine), baralgin (5 ml par voie intramusculaire ou intraveineuse), thalamonal (2-4 ml par voie intramusculaire), promedol, etc.

Lorsque des vomissements sont indiqués, la chlorpromazine, la diprazine, la diphénhydramine, le taralen, etc.. Dans le traitement des patients atteints de lithiase biliaire, un traitement chirurgical et conservateur est utilisé. Récemment, la méthode de lithotritie chimique (cholélitholytique) et extracorporelle par ondes de choc a été développée et mise en pratique.

Thérapie cholélitholytique- une méthode de traitement de la lithiase biliaire avec des médicaments capables de dissoudre les calculs biliaires. À cette fin, des préparations d'acides biliaires sont utilisées - chénodésoxycholique et ursodésoxycholique. Ils sont produits par un certain nombre de sociétés étrangères appelées henofalk et ursofalk (Allemagne), henochol (Yougoslavie), urso (Japon) et autres fonction de la vésicule biliaire et de son remplissage incomplet avec des calculs. Les petites pierres de cholestérol flottantes se prêtent mieux à la dissolution. Contre-indications: cholécystite aiguë et subaiguë, cholangite, maladie du foie, lésions inflammatoires et érosives et ulcéreuses de l'estomac, grossesse ; il est inapproprié de prescrire pour une entérocolite survenant avec un syndrome de malabsorption, ainsi qu'avec des médicaments ayant des propriétés de sorption. La dose d'acide chénodésoxycholique est de 15 mg pour 1 kg de poids corporel par jour (en 2-3 doses, mais le plus souvent la dose quotidienne entière est administrée au coucher). La durée du traitement est de 2 à 24 mois. Contrôle : ultrasons dynamiques. Si dans les 4 à 8 mois, il n'y a pas de diminution notable de la taille des calculs biliaires, un traitement ultérieur avec ce médicament n'est pas conseillé. Après la dissolution des calculs, le traitement est poursuivi pendant 3 à 6 mois supplémentaires. La dose d'acide ursodésoxycholique est de 10 mg pour 1 kg de poids corporel par jour. À l'heure actuelle, recourent plus souvent à traitement combiné acides chénodésoxycholique et ursodésoxycholique (lithofalk).

Chololithotripsie par onde de choc Il est utilisé en présence de gros calculs de cholestérol (jusqu'à 3 cm de diamètre). 2 semaines avant la cholélithotripsie et dans les 3 à 6 mois qui suivent, un traitement par l'acide chéno- et ursoséoxycholique aux doses habituelles est recommandé. S'il n'y a pas d'effet - traitement chirurgical prévu. Avec la cholécystite calculeuse, le traitement est chirurgical.

9.5. Syndrome cardiaque biliaire(cholécystite cardiaque, syndrome cholécystocoronaire, syndrome cholécystocardique) est observé dans les maladies du système biliaire (le plus souvent avec une cholécystite calculeuse compliquée). Le syndrome biliaire-cardiaque est compris comme un ensemble de troubles fonctionnels du myocarde (automatisme, excitabilité, conduction, contractilité) qui surviennent sur fond de troubles métaboliques et réflexo-toxiques dans la cholécystite. Mécanisme le développement du syndrome cardiaque et l'activité cardiaque altérée dans les maladies des voies biliaires sont considérés de manière ambiguë. La théorie du réflexe a reçu une large reconnaissance. Cependant, avec une exacerbation de la cholécystite, la douleur au cœur d'une angine de poitrine survient principalement chez les personnes atteintes maladie ischémique cœur (CHD). Dans ce cas, la pathologie biliaire contribue à la manifestation de la maladie coronarienne - une douleur d'angine de poitrine se produit. Dans la cholécystite chronique, souvent récurrente, le foie et le pancréas sont affectés et des troubles biochimiques complexes surviennent dans le corps du patient en raison d'un trouble du pigment, de l'enzyme, de l'électrolyte, des glucides, des lipides, des protéines et d'autres types de métabolisme dans le contexte de l'hypertension biliaire. et ivresse. Tout cela ensemble provoque des changements notables dans les processus métaboliques du myocarde, contribuant à l'apparition du syndrome biliaire-cardiaque. Dans le développement du syndrome cardiaque dans la cholécystite, la surcharge myocardique joue également un rôle en raison de l'apparition d'une hypertension dans la circulation systémique et pulmonaire, qui survient souvent lors d'une exacerbation de la cholécystite, notamment au moment de la colique biliaire.

Image clinique. Le syndrome cardiaque biliaire se manifeste par deux variantes cliniques principales : douloureuse (avec cardialgie) et indolore (avec arythmies cardiaques, divers Modifications de l'ECG). La douleur dans la région du cœur dans la variante douloureuse du syndrome est variée, souvent lancinante et de type angio, irradiant de l'hypochondre droit jusqu'au sternum sous la glande mammaire gauche. Les douleurs cardiaques peuvent survenir en même temps que les coliques biliaires ou les précéder. Ne s'arrête pas aux nitrates. La douleur survient souvent après avoir mangé des aliments gras et frits, ne s'accompagne pas d'une coloration émotionnelle de peur, d'anxiété et disparaît souvent après l'utilisation de cholagogues, chaleur sur l'hypochondre droit. La cardialgie dans la cholécystite calculeuse est l'une des indications du traitement chirurgical de la cholécystite. Après la chirurgie, la cardialgie disparaît généralement et seulement chez certains patients, la douleur persiste ou réapparaît en raison de la pierre laissée dans les voies biliaires, de la cholangite active, de l'exacerbation de la pancréatite, etc. La manifestation la plus courante de la variante indolore du syndrome biliaire-cardiaque est cardiaque arythmie. En plus des douleurs abdominales, les patients s'inquiètent des palpitations, des interruptions du rythme cardiaque dues à une extrasystole transitoire ou à une forme tachysystolique. fibrillation auriculaire. Une variation de la variante indolore du syndrome cardiaque peut être un affaiblissement de la contractilité myocardique, qui se manifeste cliniquement par un essoufflement, un affaiblissement du premier ton et du souffle systolique au sommet du cœur, une détérioration de l'hémodynamique intracardiaque et une diminution de l'éjection systolique du sang. L'évolution prolongée de la cholécystite s'accompagne généralement de violations graves de presque tous les éléments de l'ECG, en particulier de la dernière partie du complexe ventriculaire: dépression du segment ST, diminution d'amplitude, biphasique et de sens opposé de l'onde T dans les dérivations standard et thoracique. De tels troubles surviennent dans les deux variantes du syndrome biliaire-cardiaque, persistent pendant 1 à 2 semaines à partir du moment de la colique biliaire, puis disparaissent.

Traitement. Le traitement conservateur de la cholécystite et des troubles cardiaques comprend des analgésiques (analgine, baralgine), des médicaments antispastiques pour éliminer les coliques biliaires (atropine, platifilline, no-shpa, halidor, spazgan), des médicaments antiarythmiques (ritmilen, ritmonorm, novocainamide, nifédipine, vérapamil, obzidan, anapriline, foridon, trental, etc.), thérapie de désintoxication (sérum physiologique, mélange glucose-potassium, électrolytes, etc.), vitamines et autres en association avec antibiothérapie. Le syndrome biliaire-cardiaque est l'une des indications claires du traitement chirurgical de la cholécystite calculeuse.

9.6. Syndrome post-cholécystectomie (SCP)- un concept collectif qui comprend un complexe de troubles fonctionnels et organiques, principalement dans le système hépatobiliaire-pancréatique, survenant ou s'intensifiant après une cholécystectomie et d'autres interventions chirurgicales sur les voies biliaires. Le SEP se développe plusieurs mois (généralement un an ou plus) après la cholécystectomie et est diagnostiqué chez 5 à 25 % des patients opérés. Développement Le PCES est associé à de nombreuses causes. Les principaux sont les suivants : 1) examen pré- et peropératoire insuffisant et incomplet des patients et absence de diagnostic de calculs dans les voies biliaires, rétrécissement du canal cholédoque, rétrécissement de la papille duodénale majeure et, par conséquent, un volume incomplet de traitement chirurgical; 2) traitement chirurgical tardif de la cholécystite calculeuse après le développement de complications telles que pancréatite, hépatite, cholangite, qui subsistent après la cholécystectomie; 3) qualification insuffisante du chirurgien ou difficultés, échecs et erreurs lors de l'opération (atteinte du canal cholédoque, laissant un long moignon du canal cystique, cholédochoduodénoanastomose étroite, etc.); 4) ablation d'une vésicule biliaire fonctionnelle dans une cholécystite alithiasique ; 5) violation de la circulation de la bile et de l'hypertension biliaire ; 6) papillite duodénale sténosante, etc.

Les troubles qui se développent après une cholécystectomie et d'autres interventions chirurgicales sur les voies biliaires peuvent être divisés en un certain nombre de syndromes.

1. Modifications associées aux lésions des voies biliaires et à la violation de leur fonction :

A. Dyskinésie des voies biliaires : dyskinésie hypermotrice et évacuation accélérée de la bile ; dyskinésie hypertonique (hypertonicité) principalement de la voie cholédoque et spasme du sphincter d'Oddi ; dyskinésie hypotonique du canal cholédoque et spasme du sphincter; paralysie (hypotension) du sphincter.

B. Re-formation de pierres dans les voies biliaires : dans le choledoha ; autour de la suture; dans les conduits intrahépatiques (dans le canal dilaté du lobe droit).

B. Violations associées à un défaut de l'opération : calculs dans les voies biliaires passés inaperçus pendant l'opération ; cholécystectomie incomplète (long moignon de la voie biliaire); modifications cicatricielles (généralement avec rétrécissement) des voies biliaires ; sténose de la grande papille duodénale.

D. Lésions inflammatoires des voies biliaires et du foie :

voies biliaires communes (cholédochite) et intrahépatiques (cholangite ascendante); Hépatite chronique.

2. Prolapsus de la fonction de la vésicule biliaire (syndrome de l'absence de la vésicule biliaire): une violation de la digestion et de l'absorption des graisses dans l'intestin.

3. Troubles associés à des lésions d'autres organes digestifs :

A. Troubles de l'estomac et du duodénum: modifications des fonctions sécrétoires (plus souvent diminuées) et motrices (plus souvent augmentées) de l'estomac; gastrite, duodénite; lésions ulcéreuses de l'estomac et du duodénum (ulcères gastro-duodénaux secondaires).

B. Dommages au pancréas : pancréatite aiguë et chronique (cholépancréatite).

B. Troubles intestinaux : entérocolite ; dyskinésie intestinale; diarrhée associée au flux continu de bile dans les intestins et pancréatite; périviscérite.

4. Changements dans d'autres organes : syndrome biliaire-cardiaque ; hypertension pulmonaire.

5. Troubles métaboliques : protéines (dues à des dommages au foie), glucides (dues à des dommages au foie et au pancréas), lipides, vitamines (en particulier les vitamines liposolubles).

Tous les syndromes ne sont pas directement liés au traitement chirurgical de la cholécystite et à l'ablation de la vésicule biliaire. Ainsi, les trois derniers sont le résultat principalement d'un long parcours de la maladie du système hépatobiliaire, et non d'une intervention chirurgicale. Cependant, si le développement indépendant de maladies d'autres organes (pancréas, foie, estomac, intestins) est exclu, il n'y a pas d'autres raisons de troubles métaboliques, alors ils doivent être considérés comme une conséquence de la pathologie du système biliaire et des troubles causés par la maladie sous-jacente. Manifestations cliniques Les PCES sont variés. Caractérisé par des douleurs, une jaunisse, une cholangite, une hépatite avec le développement d'une cirrhose biliaire du foie. Il y a souvent des crampes dans le haut de l'abdomen ou dans l'hypochondre droit avec irradiation de l'épaule droite, de l'omoplate droite. La douleur est la même qu'avant l'ablation de la vésicule biliaire, ou des attaques de coliques hépatiques sont notées, accompagnées de démangeaisons, parfois de jaunisse, de décoloration des matières fécales et urine foncée. La douleur dans la plupart des cas est due à l'hypertension biliaire et au processus infectieux et inflammatoire dans les voies biliaires. Dans ce dernier cas, il y aura de la fièvre, une augmentation de la VS, une leucocytose.La violation de la circulation biliaire s'accompagne d'une hyperbilirubinémie intermittente et faible associée à une hyperenzymemie (AlAT, AsAT, GlDH, GGTP, etc.). Il peut y avoir des douleurs à la ceinture, des frissons accompagnés de fièvre. Dans la période intercritique, les patients notent des notes sourdes, souvent douleur constante dans l'hypochondre droit et l'épigastre, lourdeur au creux de l'estomac après avoir mangé, perte d'appétit, constipation, parfois nausées, vomissements, amaigrissement. Pour le diagnostic de l'hypertension biliaire, des méthodes indirectes sont principalement utilisées - le plus souvent une cholégraphie intraveineuse. La détection d'une hypertrophie du canal cholédoque est un signe d'hypertension biliaire (ne dépasse normalement pas 8-9 mm). Des informations plus précises sur les voies biliaires peuvent être obtenues en utilisant la cholangiopancréatographie rétrograde endoscopique (CPRE). La papillite duodénale sténosante, qui joue un rôle très important dans la clinique du SEP, est diagnostiquée par la duodénoscopie associée à la cholégraphie.

Traitement patients atteints de SEP est réalisée à la fois de manière conservatrice et chirurgicalement. Le traitement chirurgical est indiqué pour les modifications cicatricielles sévères de la voie cholédoque, les calculs de la voie cholédoque, les formes sévères de papillite duodénale sténosante. Le traitement conservateur comprend le régime n ° 5, qui est individualisé en fonction de la nature du PCES, de la tolérance des aliments individuels. Il prévoit une réduction de la teneur en calories (1800-2000 kcal), une teneur normale en protéines (85-95 g), une restriction des graisses (40-50 g) et des aliments contenant une grande quantité de cholestérol, une diminution des glucides facilement digestibles dans Le régime. 1,5 à 2 mois après l'opération, le régime est enrichi en fibres alimentaires (son de blé, flocons d'avoine, chou, carottes, salades, etc.).

En cas de stase biliaire, un régime lipotrope-gras n ° 5 à teneur normale en protéines, enrichi en produits protéiques lipotropes, en acides gras polyinsaturés et en vitamines, est prescrit. La pharmacothérapie doit viser à normaliser la fonction des sphincters des voies biliaires et du duodénum (cerucal, motilium, eglonil), l'adsorption des acides biliaires non conjugués (cholestyramine, bilignine, phosphalugel, alfagel, almagel, etc.), à réduire l'inflammation de la muqueuse (de-nol, venter, Vikair, etc.), suppression de la microflore intestinale pathogène (érythromycine, enterosedive, furazolidone, etc.). Lorsqu'ils sont inclus dans le processus pathologique du foie, des agents hépatoprotecteurs sont prescrits (Essentiale, lipostabil, legalen, hepatofalk, etc.), et en présence de pancréatite, des préparations enzymatiques (trienzyme, pancytrate, pancréatine, etc.), des inhibiteurs de la protéolyse ( contrical). Pour normaliser la composition chimique de la bile, les agents cholérétiques à effet cholérétique (lyobil, holosas, etc.), ainsi que le cholonerton, sont indiqués. Dans ces cas, les préparations d'acide chénodésoxycholique (henochol, chenofalk, holanorm, ursofalk, etc.) sont efficaces. De plus, si nécessaire, des agents neurotropes et sédatifs sont prescrits. La physiothérapie, la thérapie par l'exercice, les cures thermales ont un effet bénéfique. Le traitement joue un rôle important maladies concomitantes organes digestifs. Avec le développement d'un rétrécissement du canal cholédoque ou d'une papille duodénale majeure, d'une pancréatite indurative, d'une hépatocholédocholithiase, une intervention chirurgicale est nécessaire.

9.7. Tumeurs des voies biliaires peut être bénigne ou maligne.

9.7.1. tumeurs bénignes. Dans les voies biliaires principales se produisent très rarement adénome, fibrome ou carcinoïde. Ils se développent lentement, ne se désintègrent pas, mais peuvent entraîner un rétrécissement brutal de la lumière du conduit et une jaunisse obstructive. Il est difficile de distinguer les tumeurs bénignes du cancer par les manifestations cliniques et même au cours de la chirurgie.

9.7.2. Tumeurs malignes. Dans les voies biliaires, le cancer et très rarement le sarcome se produisent.

cancer des voies biliaires - Il s'agit d'une tumeur à croissance lente ou rapide, entraînant un ictère obstructif. Sa source est l'épithélium recouvrant la membrane muqueuse. Dominé adénocarcinomes. Arrive rarement carcinome squameux. La tumeur se développe intra-muros autour de la circonférence du conduit sous la forme d'un nœud. Il y a d'abord une obstruction partielle, puis complète du conduit. Métastases à régional Les ganglions lymphatiques, au foie, au pancréas, aux poumons et aux os.

Image clinique. L'ictère est le principal signe clinique du cancer des voies biliaires extrahépatiques. Il se développe souvent sans syndrome douloureux préalable, augmente progressivement et s'accompagne d'un prurit prononcé. Avec l'effondrement de la tumeur, une diminution temporaire de la jaunisse est possible. Simultanément à la jaunisse, l'appétit diminue, la faiblesse, la perte de poids et la constipation augmentent. Les selles se décolorent. Le foie est hypertrophié, le symptôme de Courvoisier est souvent fixe. fibrosarcome selon les caractéristiques du flux et manifestation clinique ne diffère pas du cancer et, en règle générale, n'est détecté qu'après un examen histologique.

Diagnostic. Chez les patients atteints d'ictère obstructif, le taux de bilirubine et d'acides biliaires augmente dans l'urine et l'urobiline est absente. Il y a une teneur élevée en bilirubine dans le plasma sanguin. Reconnaissance tumeurs bénignes, cancer et sarcome des voies biliaires est facilitée par la tomodensitométrie Au cours de la chirurgie, une aide essentielle pour déterminer la nature processus pathologique dans les voies biliaires peut subir une cholangiographie. Les ganglions lymphatiques régionaux sont situés près du canal cholédoque et cholédoque, des portes du foie, de la tête du pancréas, du duodénum, ​​des artères porte, coeliaque et mésentérique supérieure.

Traitement. Dans les tumeurs bénignes et malignes, l'ablation chirurgicale de la tumeur est réalisée par résection du conduit ou résection pancréatoduodénale.

cancer de la vésicule biliaire. C'est rare tumeur maligne, qui survient principalement chez les femmes de plus de 40 ans et se développe souvent dans un contexte de lithiase biliaire à long terme et d'inflammation chronique de la vésicule biliaire. Le carcinome survient dans n'importe quelle partie de la vésicule biliaire, se développe sous la forme d'un nœud ou d'une formation diffuse, parfois multicentrique. Avec un taux de croissance rapide, il y a souvent une ulcération et une désintégration de la tumeur, entraînant des saignements chroniques. La propagation du cancer au canal cystique entraîne une hydropisie de la vessie. Histologiquement isolé adénocarcinome, scirr et cancer médullaire.

Image clinique. Principal signes cliniques cancer de la vésicule biliaire sont des douleurs dans l'hypochondre droit, une perte d'appétit, une faiblesse, une perte de poids, une jaunisse obstructive. Ce dernier indique la propagation du cancer aux voies biliaires et au foie. Diagnostic. Le cancer de la vésicule biliaire est souvent détecté par palpation. Dans l'hypochondre droit, une tumeur dense, tubéreuse et non déplaçable est déterminée. La reconnaissance du cancer dans une période antérieure de la maladie est facilitée par l'échographie, la tomodensitométrie, la cholangiographie percutanée et intraveineuse, la laparoscopie et l'angiographie sélective. Les ganglions lymphatiques régionaux sont situés près du canal cholédoque et cholédoque, des portes du foie, près de la tête du pancréas, du duodénum, ​​des vaisseaux coeliaques portails et mésentériques supérieurs.

Traitement. Ablation chirurgicale vésicule biliaire, parfois avec résection du lobe carré du foie. Prévoir mauvais, car la dissémination se produit tôt cellules cancéreuses par les vaisseaux lymphatiques.

Date d'ajout : 2014-12-12 | Vues : 4388 | violation de copyright


| | | | | | | | | | | | | 14 | | | | | | | | | |